Классика        08.11.2020   

Можем составить уравнение. Прямая линия. Уравнение прямой. Как составить уравнение по условию задачи

Химия – это наука о веществах, их свойствах и превращениях .
То есть, если с окружающими нас веществами ничего не происходит, то это не относится к химии. Но что значит, «ничего не происходит»? Если в поле нас вдруг застала гроза, и мы все промокли, как говорится «до нитки», то это ли не превращение: ведь одежда была сухой, а стала мокрой.

Если, к примеру взять железный гвоздь, обработать его напильником, а затем собрать железные опилки (Fe ) , то это ли так же не превращение: был гвоздь – стал порошок. Но если после этого собрать прибор и провести получение кислорода (О 2) : нагреть перманганат калия (КМпО 4) и собрать в пробирку кислород, а затем в неё поместить раскалённые «до красна» эти железные опилки, то они вспыхнут ярким пламенем и после сгорания превратятся в порошок бурого цвета. И это так же превращение. Так где же химия? Несмотря на то, что в этих примерах меняется форма (железный гвоздь) и состояние одежды (сухая, мокрая) – это не превращения. Дело в том, что сам по себе гвоздь как был веществом (железо), так им и остался, несмотря на другую свою форму, а воду от дождя как впитала наша одежда, так потом его и испарила в атмосферу. Сама вода не изменилась. Так что же такое превращения с точки зрения химии?

Превращениями с точки зрения химии называются такие явления, которые сопровождаются изменением состава вещества. Возьмём в качестве примера тот же гвоздь. Не важно, какую форму он принял после обработки напильником, но после того как собранные от него железные опилки поместили в атмосферу кислорода - он превратился в оксид железа (Fe 2 O 3 ) . Значит, что-то всё-таки изменилось? Да, изменилось. Было вещество гвоздь, но под воздействием кислорода сформировалось новое вещество – оксид элемента железа. Молекулярное уравнение этого превращения можно отобразить следующими химическими символами:

4Fe + 3O 2 = 2Fe 2 O 3 (1)

Для непосвящённого в химии человека сразу возникают вопросы. Что такое «молекулярное уравнение», что такое Fe? Почему поставлены цифры «4», «3», «2»? Что такое маленькие цифры «2» и «3» в формуле Fe 2 O 3 ? Это значит, наступило время во всём разобраться по порядку.

Знаки химических элементов.

Несмотря на то, что химию начинают изучать в 8-м классе, а некоторые даже раньше, многим известен великий русский химик Д. И. Менделеев. И конечно же, его знаменитая «Периодическая система химических элементов». Иначе, проще, её называют «Таблица Менделеева».

В этой таблице, в соответствующем порядке, располагаются элементы. К настоящему времени их известно около 120. Названия многих элементов нам были известны ещё давно. Это: железо, алюминий, кислород, углерод, золото, кремний. Раньше мы не задумываясь применяли эти слова, отождествляя их с предметами: железный болт, алюминиевая проволока, кислород в атмосфере, золотое кольцо и т.д. и т.д. Но на самом деле все эти вещества (болт, проволока, кольцо) состоят из соответствующих им элементов. Весь парадокс состоит в том, что элемент нельзя потрогать, взять в руки. Как же так? В таблице Менделеева они есть, а взять их нельзя! Да, именно так. Химический элемент – это абстрактное (то есть отвлечённое) понятие, и используется в химии, впрочем как и в других науках, для расчётов, составления уравнений, при решении задач. Каждый элемент отличается от другого тем, что для него характерна своя электронная конфигурация атома. Количество протонов в ядре атома равно количеству электронов в его орбиталях. К примеру, водород – элемент №1. Его атом состоит из 1-го протона и 1-го электрона. Гелий – элемент №2. Его атом состоит из 2-х протонов и 2-х электронов. Литий – элемент №3. Его атом состоит из 3-х протонов и 3-х электронов. Дармштадтий – элемент №110. Его атом состоит из 110-и протонов и 110-и электронов.

Каждый элемент обозначается определённым символом, латинскими буквами, и имеет определённое прочтение в переводе с латинского. Например, водород имеет символ «Н» , читается как «гидрогениум» или «аш». Кремний имеет символ «Si» читается как «силициум». Ртуть имеет символ «Нg» и читается как «гидраргирум». И так далее. Все эти обозначения можно найти в любом учебнике химии за 8-й класс. Для нас сейчас главное уяснить то, что при составлении химических уравнений, необходимо оперировать указанными символами элементов.

Простые и сложные вещества.

Обозначая единичными символами химических элементов различные вещества (Hg ртуть , Fe железо , Cu медь , Zn цинк , Al алюминий ) мы по сути обозначаем простые вещества, то есть вещества, состоящие из атомов одного вида (содержащие одно и то же количество протонов и нейтронов в атоме). Например, если во взаимодействие вступают вещества железо и сера, то уравнение примет следующую форму записи:

Fe + S = FeS (2)

К простым веществам относятся металлы (Ва, К, Na, Mg, Ag), а так же неметаллы (S, P, Si, Cl 2 , N 2 , O 2 , H 2). Причём следует обратить
особое внимание на то, что все металлы обозначаются единичными символами: К, Ва, Са, Аl, V, Mg и т.д., а неметаллы – либо простыми символами: C,S,P или могут иметь различные индексы, которые указывают на их молекулярное строение: H 2 , Сl 2 , О 2 , J 2 , P 4 , S 8 . В дальнейшем это будет иметь очень большое значение при составлении уравнений. Совсем не трудно догадаться, что сложными веществами являются вещества, образованные из атомов разного вида, например,

1). Оксиды:
оксид алюминия Al 2 O 3 ,

оксид натрия Na 2 O,
оксид меди CuO,
оксид цинка ZnO,
оксид титана Ti 2 O 3 ,
угарный газ или оксид углерода (+2) CO,
оксид серы (+6) SO 3

2). Основания:
гидроксид железа (+3) Fe(OH) 3 ,
гидроксид меди Cu(OH) 2 ,
гидроксид калия или щёлочь калия КOH,
гидроксид натрия NaOH.

3). Кислоты:
соляная кислота HCl,
сернистая кислота H 2 SO 3 ,
азотная кислота HNO 3

4). Соли:
тиосульфат натрия Na 2 S 2 O 3 ,
сульфат натрия или глауберова соль Na 2 SO 4 ,
карбонат кальция или известняк СаCO 3,
хлорид меди CuCl 2

5). Органические вещества:
ацетат натрия СН 3 СООNa,
метан СН 4 ,
ацетилен С 2 Н 2 ,
глюкоза С 6 Н 12 О 6

Наконец, после того как мы выяснили структуру различных веществ, можно приступать к составлению химических уравнений.

Химическое уравнение.

Само слово «уравнение» производное от слова «уравнять», т.е. разделить нечто на равные части. В математике уравнения составляют чуть ли не самую сущность этой науки. К примеру, можно привести такое простое уравнение, в котором левая и правая части будут равны «2»:

40: (9 + 11) = (50 х 2) : (80 – 30);

И в химических уравнениях тот же принцип: левая и правая части уравнения должны соответствовать одинаковым количествам атомов, участвующим в них элементов. Или, если приводится ионное уравнение, то в нём число частиц так же должно соответствовать этому требованию. Химическим уравнением называется условная запись химической реакции с помощью химических формул и математических знаков. Химическое уравнение по своей сути отражает ту или иную химическую реакцию, то есть процесс взаимодействия веществ, в процессе которых возникают новые вещества. Например, необходимо написать молекулярное уравнение реакции, в которой принимают участие хлорид бария ВаСl 2 и серная кислота H 2 SO 4. В результате этой реакции образуется нерастворимый осадок – сульфат бария ВаSO 4 и соляная кислота НСl:

ВаСl 2 + H 2 SO 4 = BaSO 4 + 2НСl (3)

Прежде всего необходимо уяснить, что большая цифра «2», стоящая перед веществом НСlназывается коэффициентом, а малые цифры «2», «4» под формулами ВаСl 2 , H 2 SO 4 ,BaSO 4 называются индексами. И коэффициенты и индексы в химических уравнениях выполняют роль множителей, а не слагаемых. Что бы правильно записать химическое уравнение, необходимо расставить коэффициенты в уравнении реакции . Теперь приступим к подсчёту атомов элементов в левой и правой частях уравнения. В левой части уравнения: в веществе ВаСl 2 содержатся 1 атом бария (Ва), 2 атома хлора (Сl). В веществе H 2 SO 4: 2 атома водорода (Н), 1 атом серы (S) и 4 атома кислорода (О) . В правой части уравнения: в веществе BaSO 4 1 атом бария (Ва) 1 атом серы (S) и 4 атома кислорода (О), в веществе НСl: 1 атом водорода (Н) и 1 атом хлора (Сl). Откуда следует, что в правой части уравнения количество атомов водорода и хлора вдвое меньше, чем в левой части. Следовательно, перед формулой НСl в правой части уравнения необходимо поставить коэффициент «2». Если теперь сложить количества атомов элементов, участвующих в данной реакции, и слева и справа, то получим следующий баланс:

В обеих частях уравнения количества атомов элементов, участвующих в реакции, равны, следовательно оно составлено правильно.

Химические уравнение и химические реакции

Как мы уже выяснили, химические уравнения являются отражением химических реакций. Химическими реакциями называются такие явления, в процессе которых происходит превращение одних веществ в другие. Среди их многообразия можно выделить два основных типа:

1). Реакции соединения
2). Реакции разложения.

В подавляющем своём большинстве химические реакции принадлежат к реакциям присоединения, поскольку с отдельно взятым веществом редко могут происходить изменения в его составе, если оно не подвергается воздействиям извне (растворению, нагреванию, действию света). Ничто так не характеризует химическое явление, или реакцию, как изменения, происходящие при взаимодействии двух и более веществ. Такие явления могут осуществляться самопроизвольно и сопровождаться повышением или понижением температуры, световыми эффектами, изменением цвета, образованием осадка, выделением газообразных продуктов, шумом.

Для наглядности приведём несколько уравнений, отражающих процессы реакций соединения, в процессе которых получаются хлорид натрия (NaCl), хлорид цинка (ZnCl 2), осадок хлорида серебра (AgCl), хлорид алюминия (AlCl 3)

Cl 2 + 2Nа = 2NaCl (4)

СuCl 2 + Zn= ZnCl 2 + Сu (5)

AgNO 3 + КCl = AgCl + 2KNO 3 (6)

3HCl + Al(OH) 3 = AlCl 3 + 3Н 2 О (7)

Cреди реакций соединения следует особым образом отметить следующие: замещения (5), обмена (6), и как частный случай реакции обмена – реакцию нейтрализации (7).

К реакциям замещения относятся такие, при осуществлении которой атомы простого вещества замещают атомы одного из элементов в сложном веществе. В примере (5) атомы цинка замещают из раствора СuCl 2 атомы меди, при этом цинк переходит в растворимую соль ZnCl 2 , а медь выделяется из раствора в металлическом состоянии.

К реакциям обмена относятся такие реакции, при которых два сложных вещества обмениваются своими составными частями. В случае реакции (6) растворимые соли AgNO 3 и КCl при сливании обоих растворов образуют нерастворимый осадок соли AgCl. При этом они обмениваются своими составными частями – катионами и анионами. Катионы калия К + присоединяются к анионам NO 3 , а катионы серебра Ag + – к анионам Cl - .

К особому, частному случаю, реакций обмена относится реакция нейтрализации. К реакциям нейтрализации относятся такие реакции, в процессе которых кислоты реагируют с основаниями, в результате образуется соль и вода. В примере (7) соляная кислота HCl , реагируя с основанием Al(OH) 3 образует соль AlCl 3 и воду. При этом катионы алюминия Al 3+ от основания обмениваются с анионами Сl - от кислоты. В итоге происходит нейтрализация соляной кислоты.

К реакциям разложения относятся такие, при котором из одного сложного образуются два и более новых простых или сложных веществ, но более простого состава. В качестве реакций можно привести такие, в процессе которых разлагаются 1). Нитрат калия (КNO 3) с образованием нитрита калия (КNO 2) и кислорода (O 2); 2). Перманганат калия (KMnO 4): образуются манганат калия (К 2 МnO 4), оксид марганца (MnO 2) и кислород (O 2); 3). Карбонат кальция или мрамор ; в процессе образуются углекислый газ (CO 2) и оксид кальция (СаО)

2КNO 3 = 2КNO 2 + O 2 (8)
2KMnO 4 = К 2 МnO 4 + MnO 2 + O 2 (9)
СаCO 3 = CaO + CO 2 (10)

В реакции (8) из сложного вещества образуется одно сложное и одно простое. В реакции (9) – два сложных и одно простое. В реакции (10) – два сложных вещества, но более простых по составу

Разложению подвергаются все классы сложных веществ:

1). Оксиды: оксид серебра 2Ag 2 O = 4Ag + O 2 (11)

2). Гидроксиды: гидроксид железа 2Fe(OH) 3 = Fe 2 O 3 + 3H 2 O (12)

3). Кислоты: серная кислота H 2 SO 4 = SO 3 + H 2 O (13)

4). Соли: карбонат кальция СаCO 3 = СаO + CO 2 (14)

5). Органические вещества: спиртовое брожение глюкозы

С 6 Н 12 О 6 = 2С 2 Н 5 ОH + 2CO 2 (15)

Согласно другой классификации, все химические реакции можно разделить на два типа: реакции, идущие с выделением теплоты, их называют экзотермические, и реакции, идущие с поглощением теплоты – эндотермические. Критерием таких процессов является тепловой эффект реакции. Как правило, к экзотермическим реакциям относятся реакции окисления, т.е. взаимодействия с кислородом, например сгорание метана :

СН 4 + 2O 2 = СО 2 + 2Н 2 О + Q (16)

а к эндотермическим реакциям – реакции разложения, уже приводимые выше (11) – (15). Знак Q в конце уравнения указывает на то, выделяется ли теплота в процессе реакции (+Q) или поглощается (-Q):

СаCO 3 = СаO+CO 2 - Q (17)

Можно так же рассматривать все химические реакции по типу изменения степени окисления, участвующих в их превращениях элементов. К примеру, в реакции (17) участвующие в ней элементы не меняют свои степени окисления:

Са +2 C +4 O 3 -2 = Са +2 O -2 +C +4 O 2 -2 (18)

А в реакции (16) элементы меняют свои степени окисления:

2Mg 0 + O 2 0 = 2Mg +2 O -2

Реакции такого типа относятся к окислительно-восстановительным . Они будут рассматриваться отдельно. Для составления уравнений по реакциям такого типа необходимо использовать метод полуреакций и применять уравнение электронного баланса.

После приведения различных типов химических реакций, можно приступать к принципу составлений химических уравнений, иначе, подбору коэффициентов в левой и правой их частях.

Механизмы составления химических уравнений.

К какому бы типу ни относилась та или иная химическая реакция, её запись (химическое уравнение) должна соответствовать условию равенства количества атомов до реакции и после реакции.

Существуют такие уравнения (17), которые не требуют уравнивания, т.е. расстановки коэффициентов. Но в большинстве случаях, как в примерах (3), (7), (15), необходимо предпринимать действия, направленные на уравнивание левой и правой частей уравнения. Какими же принципами необходимо руководствоваться в таких случаях? Существует ли какая ни будь система в подборе коэффициентов? Существует, и не одна. К таковым системам относятся:

1). Подбор коэффициентов по заданным формулам.

2). Составление по валентностям реагирующих веществ.

3). Составление по степеням окисления реагирующих веществ.

В первом случае полагается, что нам известны формулы реагирующих веществ как до реакции, так и после. К примеру, дано следующее уравнение:

N 2 + О 2 →N 2 О 3 (19)

Принято считать, что пока не установлено равенство между атомами элементов до реакции и после, знак равенства (=) в уравнении не ставится, а заменяется стрелкой (→). Теперь приступим к собственно уравниванию. В левой части уравнения имеются 2 атома азота (N 2) и два атома кислорода (О 2), а в правой – два атома азота (N 2) и три атома кислорода (О 3). По количеству атомов азота его уравнивать не надо, но по кислороду необходимо добиться равенства, поскольку до реакции их участвовало два атома, а после реакции стало три атома. Составим следующую схему:

до реакции после реакции
О 2 О 3

Определим наименьшее кратное между данными количествами атомов, это будет «6».

О 2 О 3
\ 6 /

Разделим это число в левой части уравнения по кислороду на «2». Получим число «3», поставим его в решаемое уравнение:

N 2 + 3О 2 →N 2 О 3

Так же разделим число «6» для правой части уравнения на «3». Получим число «2», так же поставим его в решаемое уравнение:

N 2 + 3О 2 → 2N 2 О 3

Количества атомов кислорода и в левой и в правой частях уравнения стали равны, соответственно по 6 атомов:

Но количество атомов азота в обеих частях уравнения не будут соответствовать друг другу:

В левой – два атома, в правой – четыре атома. Следовательно, что бы добиться равенства, необходимо удвоить количество азота в левой части уравнения, поставив коэффициент «2»:

Таким образом, равенство по азоту соблюдено и в целом, уравнение примет вид:

2N 2 + 3О 2 → 2N 2 О 3

Теперь в уравнении можно вместо стрелки поставит знак равенства:

2N 2 + 3О 2 = 2N 2 О 3 (20)

Приведём другой пример. Дано следующее уравнение реакции:

Р + Cl 2 → РCl 5

В левой части уравнения имеется 1 атом фосфора (Р) и два атома хлора (Cl 2), а в правой – один атом фосфора (Р) и пять атомов кислорода (Cl 5). По количеству атомов фосфора его уравнивать не надо, но по хлору необходимо добиться равенства, поскольку до реакции их участвовало два атома, а после реакции стало пять атома. Составим следующую схему:

до реакции после реакции
Cl 2 Cl 5

Определим наименьшее кратное между данными количествами атомов, это будет «10».

Cl 2 Cl 5
\ 10 /

Разделим это число в левой части уравнения по хлору на «2». Получим число «5», поставим его в решаемое уравнение:

Р + 5Cl 2 → РCl 5

Так же разделим число «10» для правой части уравнения на «5». Получим число «2», так же поставим его в решаемое уравнение:

Р + 5Cl 2 → 2РCl 5

Количества атомов хлора и в левой и в правой частях уравнения стали равны, соответственно по 10 атомов:

Но количество атомов фосфора в обеих частях уравнения не будут соответствовать друг другу:

Следовательно, что бы добиться равенства, необходимо удвоить количество фосфора в левой части уравнения, поставив коэффициент «2»:

Таким образом, равенство по фосфору соблюдено и в целом, уравнение примет вид:

2Р + 5Cl 2 = 2РCl 5 (21)

При составлении уравнений по валентностям необходимо дать определение валентности и установить значения для наиболее известных элементов. Валентность – это одно из ранее применяемых понятий, в настоящее время в ряде школьных программ не используется. Но при его помощи легче объяснить принципы составления уравнений химических реакций. Под валентностью понимают число химических связей, которые тот или иной атом может образовывать с другим, или другими атомами . Валентность не имеет знака (+ или -) и обозначается римскими цифрами, как правило, над символами химических элементов, например:

Откуда берутся эти значения? Как их применять при составлении химических уравнений? Числовые значения валентностей элементов совпадают с их номером группы Периодической системы химических элементов Д. И. Менделеева (Таблица 1).

Для других элементов значения валентностей могут иметь иные значения, но никогда не больше номера группы, в которой они расположены. Причём для чётных номеров групп (IV и VI) валентности элементов принимают только чётные значения, а для нечётных – могут иметь как чётные, так и нечётные значения (Таблица.2).

Конечно же, в значениях валентностей для некоторых элементов имеются исключения, но в каждом конкретном случае эти моменты обычно оговариваются. Теперь рассмотрим общий принцип составления химических уравнений по заданным валентностям для тех или иных элементов. Чаще всего данный метод приемлем в случае составления уравнений химических реакций соединения простых веществ, например, при взаимодействии с кислородом (реакции окисления ). Допустим, необходимо отобразить реакцию окисления алюминия . Но напомним, что металлы обозначаются единичными атомами (Al), а неметаллы, находящиеся в газообразном состоянии – с индексами «2» - (О 2). Сначала напишем общую схему реакции:

Al + О 2 →AlО

На данном этапе ещё не известно, какое правильное написание должно быть у оксида алюминия. И вот именно на данном этапе нам на помощь придёт знание валентностей элементов. Для алюминия и кислорода проставим их над предполагаемой формулой этого оксида:

III II
Al О

После чего «крест»-на-«крест» у этих символов элементов поставим внизу соответствующие индексы:

III II
Al 2 О 3

Состав химического соединения Al 2 О 3 определён. Дальнейшая схема уравнения реакции примет вид:

Al+ О 2 →Al 2 О 3

Остаётся только уравнять левую и правую его части. Поступим таким же способом, как в случае составления уравнения (19). Количества атомов кислорода уравняем, прибегая к нахождению наименьшего кратного:

до реакции после реакции

О 2 О 3
\ 6 /

Разделим это число в левой части уравнения по кислороду на «2». Получим число «3», поставим его в решаемое уравнение. Так же разделим число «6» для правой части уравнения на «3». Получим число «2», так же поставим его в решаемое уравнение:

Al + 3О 2 → 2Al 2 О 3

Что бы добиться равенства по алюминию, необходимо скорректировать его количество в левой части уравнения, поставив коэффициент «4»:

4Al + 3О 2 → 2Al 2 О 3

Таким образом, равенство по алюминию и кислороду соблюдено и в целом, уравнение примет окончательный вид:

4Al + 3О 2 = 2Al 2 О 3 (22)

Применяя метод валентностей, можно прогнозировать, какое вещество образуется в процессе химической реакции, как будет выглядеть его формула. Допустим, в реакцию соединения вступили азот и водород с соответствующими валентностями III и I. Напишем общую схему реакции:

N 2 + Н 2 → NН

Для азота и водорода проставим валентности над предполагаемой формулой этого соединения:

Как и прежде «крест»-на-«крест» у этих символов элементов поставим внизу соответствующие индексы:

III I
N Н 3

Дальнейшая схема уравнения реакции примет вид:

N 2 + Н 2 → NН 3

Уравнивая уже известным способом, через наименьшее кратное для водорода, равное «6»,получим искомые коэффициенты, и уравнение в целом:

N 2 + 3Н 2 = 2NН 3 (23)

При составлении уравнений по степеням окисления реагирующих веществ необходимо напомнить, что степенью окисления того или иного элемента называется число принятых или отданных в процессе химической реакции электронов. Степень окисления в соединениях в основном, численно совпадает со значениями валентностей элемента. Но отличаются знаком. Например, для водорода валентность равна I, а степень окисления (+1) или (-1). Для кислорода валентность равна II, а степень окисления (-2). Для азота валентности равны I,II,III,IV,V, а степени окисления (-3), (+1), (+2), (+3), (+4), (+5) и т.д. Степени окисления наиболее часто применяемых в уравнениях элементов, приведены в таблице 3.

В случае реакций соединения принцип составления уравнений по степеням окисления такой же, как и при составлении по валентностям. Например, приведём уравнение реакции окисления хлора кислородом, в которой хлор образует соединение со степенью окисления +7. Запишем предполагаемое уравнение:

Cl 2 + О 2 → ClО

Поставим над предполагаемым соединением ClО степени окисления соответствующих атомов:

Как и в предыдущих случаях установим, что искомая формула соединения примет вид:

7 -2
Cl 2 О 7

Уравнение реакции примет следующий вид:

Cl 2 + О 2 → Cl 2 О 7

Уравнивая по кислороду, найдя наименьшее кратное между двумя и семи, равное «14», установим в итоге равенство:

2Cl 2 + 7О 2 = 2Cl 2 О 7 (24)

Несколько иной способ необходимо применять со степенями окисления при составлении реакций обмена, нейтрализации, замещения. В ряде случаев предоставляется затруднительным узнать: какие соединения образуются при взаимодействии сложных веществ?

Как узнать: что получится в процессе реакции?

Действительно, как узнать: какие продукты реакции могут возникнут в ходе конкретной реакции? К примеру, что образуется при взаимодействии нитрата бария и сульфата калия?

Ва(NО 3) 2 + К 2 SO 4 → ?

Может быть ВаК 2 (NО 3) 2 + SO 4 ? Или Ва + NО 3 SO 4 + К 2 ? Или ещё что-то? Конечно же, в процессе этой реакции образуются соединения: ВаSO 4 и КNО 3 . А откуда это известно? И как правильно написать формулы веществ? Начнём с того, что чаще всего упускается из вида: с самого понятия «реакция обмена». Это значит, что при данных реакциях вещества меняются друг с другом составными частями. Поскольку реакции обмена в большинстве своём осуществляются межу основаниями, кислотами или солями, то частями, которыми они будут меняться, являются катионы металлов (Na + , Mg 2+ ,Al 3+ ,Ca 2+ ,Cr 3+), ионов Н + или ОН - , анионов – остатков кислот, (Cl - , NO 3 2- ,SO 3 2- , SO 4 2- , CO 3 2- , PO 4 3-). В общем виде реакцию обмена можно привести в следующей записи:

Kt1An1 + Kt2An1 = Kt1An2 + Kt2An1 (25)

Где Kt1 и Kt2 – катионы металлов (1) и (2), а An1 и An2 – соответствующие им анионы (1) и (2). При этом обязательно надо учитывать, что в соединениях до реакции и после реакции на первом месте всегда устанавливаются катионы, а анионы – на втором. Следовательно, если в реакцию вступит хлорид калия и нитрат серебра , оба в растворённом состоянии

KCl + AgNO 3 →

то в процессе её образуются вещества KNO 3 и AgClи соответствующее уравнение примет вид:

KCl + AgNO 3 =KNO 3 + AgCl (26)

При реакциях нейтрализации протоны от кислот (Н +) будут соединяться с анионами гидроксила (ОН -) с образованием воды (Н 2 О):

НCl + КОН = КCl + Н 2 O (27)

Степени окисления катионов металлов и заряды анионов кислотных остатков указаны в таблице растворимости веществ (кислот, солей и оснований в воде). По горизонтали приведены катионы металлов, а по вертикали – анионы кислотных остатков.

Исходя из этого, при составлении уравнения реакции обмена, необходимо вначале в левой его части установить степени окисления принимающих в этом химическом процессе частиц. Например, требуется написать уравнение взаимодействия между хлоридом кальция и карбонатом натрия.Составим исходную схему этой реакции:

СаCl + NаСО 3 →

Са 2+ Cl - + Nа + СО 3 2- →

Совершив уже известное действие «крест»-на-«крест», определим реальные формулы исходных веществ:

СаCl 2 + Nа 2 СО 3 →

Исходя из принципа обмена катионами и анионами (25), установим предварительные формулы образующихся в ходе реакции веществ:

СаCl 2 + Nа 2 СО 3 → СаСО 3 + NаCl

Над их катионами и анионами проставим соответствующие заряды:

Са 2+ СО 3 2- + Nа + Cl -

Формулы веществ записаны правильно, в соответствии с зарядами катионов и анионов. Составим полное уравнение, уравняв левую и правую его части по натрию и хлору:

СаCl 2 + Nа 2 СО 3 = СаСО 3 + 2NаCl (28)

В качестве другого примера приведём уравнение реакции нейтрализации между гидроксидом бария и ортофосфорной кислотой:

ВаОН + НРО 4 →

Над катионами и анионами проставим соответствующие заряды:

Ва 2+ ОН - + Н + РО 4 3- →

Определим реальные формулы исходных веществ:

Ва(ОН) 2 + Н 3 РО 4 →

Исходя из принципа обмена катионами и анионами (25), установим предварительные формулы образующихся в ходе реакции веществ, учитывая, что при реакции обмена одним из веществ обязательно должна быть вода:

Ва(ОН) 2 + Н 3 РО 4 → Ва 2+ РО 4 3- + Н 2 O

Определим правильную запись формулы соли, образовавшейся в процессе реакции:

Ва(ОН) 2 + Н 3 РО 4 → Ва 3 (РО 4) 2 + Н 2 O

Уравняем левую часть уравнения по барию:

3Ва (ОН) 2 + Н 3 РО 4 → Ва 3 (РО 4) 2 + Н 2 O

Поскольку в правой части уравнения остаток ортофосфорной кислоты взят дважды, (РО 4) 2 , то слева необходимо также удвоить её количество:

3Ва (ОН) 2 + 2Н 3 РО 4 → Ва 3 (РО 4) 2 + Н 2 O

Осталось привести в соответствие количество атомов водорода и кислорода в правой части у воды. Так как слева общее количество атомов водорода равно 12, то справа оно так же должно соответствовать двенадцати, поэтому перед формулой воды необходимо поставить коэффициент «6» (поскольку в молекуле воды уже имеется 2 атома водорода). По кислороду так же соблюдено равенство: слева 14 и справа 14. Итак, уравнение имеет правильную форму записи:

3Ва (ОН) 2 + 2Н 3 РО 4 → Ва 3 (РО 4) 2 + 6Н 2 O (29)

Возможность осуществления химических реакций

Мир состоит из великого множества веществ. Неисчислимо так же количество вариантов химических реакций между ними. Но можем ли мы, написав на бумаге то или иное уравнение утверждать, что ему будет соответствовать химическая реакция? Существует ошибочное мнение, что если правильно расставить коэффициенты в уравнении, то оно будет осуществимо и на практике. Например, если взять раствор серной кислоты и опустить в него цинк , то можно наблюдать процесс выделения водорода:

Zn+ H 2 SO 4 = ZnSO 4 + H 2 (30)

Но если в этот же раствор опустить медь, то процесс выделения газа наблюдаться не будет. Реакция не осуществима.

Cu+ H 2 SO 4 ≠

В случае, если будет взята концентрированная серная кислота, она будет реагировать с медью:

Cu + 2H 2 SO 4 = CuSO 4 + SO 2 + 2Н 2 O (31)

В реакции (23) между газами азотом и водородом наблюдается термодинамическое равновесие, т.е. сколько молекул аммиака NН 3 образуется в единицу времени, столько же их и распадётся обратно на азот и водород. Смещение химического равновесия можно добиться повышением давления и понижением температуры

N 2 + 3Н 2 = 2NН 3

Если взять раствор гидроксида калия и прилить к нему раствор сульфата натрия , то никаких изменений наблюдаться не будет, реакция будет не осуществима:

КОН + Na 2 SO 4 ≠

Раствор хлорида натрия при взаимодействии с бромом не будет образовывать бром, несмотря на то, что данная реакция может быть отнесена к реакции замещения:

NаCl + Br 2 ≠

В чём же причины таких несоответствий? Дело в том, что оказывается недостаточно только правильно определять формулы соединений , необходимо знать специфику взаимодействия металлов с кислотами, умело пользоваться таблицей растворимости веществ, знать правила замещения в ряду активности металлов и галогенов. В этой статье излагаются только самые основные принципы как расставить коэффициенты в уравнениях реакций , как написать молекулярные уравнения , как определить состав химического соединения.

Химия, как наука, чрезвычайно разнообразна и многогранна. В приведённой статье отражена лишь малая часть процессов, происходящих в реальном мире. Не рассмотрены типы , термохимические уравнения, электролиз, процессы органического синтеза и многое, многое другое. Но об этом в следующих статьях.

сайт, при полном или частичном копировании материала ссылка на первоисточник обязательна.

ОТДЕЛЕНИЕ VI.

ПРЕОБРАЗОВАНИЯ РАВЕНСТВ.

___________

РЕШЕНИЕ И СОСТАВЛЕНИЕ УРАВНЕНИЙ 1-Й СТЕПЕНИ

§ 5. Составление уравнения c одним неизвстным.

Всякая арифметическая задача состоит в том, что по нeскольким извeстным величинам и по данным соотношениям между этими извeстными величинами и другими, неизвeстными, отыскиваются нeизвeстные. Алгебра дает особый способ для рeшения арифмети-ческих задач. Этот способ основан на том, что словесно выраженные условия арифметических задач могут быть переводимы на алгебраический язык, т.е. выражаемы посредством алгебраичeских формул.

Перевод словeсно выражeнных условий задачи на алгeбраический язык вообщe называeтся составлением формул.

Составить по условиям задачи уравнение с одним неизвeстным значит так перевести эти условия на алгебраический язык, чтобы вся совокупность этих условий выразилась одним уравнением, содeржащим одно неизвeстное. Для этого необходимо, чтобы число отдeльных независимых между собою условий задачи было бы равно числу подразумeваемых в ней неизвeстных.

Вслeдствиe чрезвычайного разнообразия задач приемы составления уравнений, соотвeтствующих этим задачам, чрезвычайно разнообразны. Общих правил для составления уравнений нeт. Но есть одно общеe указание, которое руководит нашим рассуждением при переводe условий задачи на алгебраический язык и позволяет нам с самаго начала рассуждения идти вeрным путем к достижению окончательной цeли. Это общее указание, или общий принцип составлeиия уравнения мы выразим слeдующим образом:

Чтобы составить по условиям задачи уравнение с одним неизвeстным, нужно:

1) выбрать между неизвeстными, которые в задачe или прямо указываются, или подразумeваются, какое-нибудь одно, принимаемое за первое, и обозначить это неизвeстноe какой-нибудь буквой, напр., х ;

2) посредством этого обозначения и обозначений, данных в задачe, выразить всe величины, о которых в задачe прямо говорится, или которые подразумeваются, наблюдая, чтобы при составлении таких выражений постепенно принимались во внимание всe данные в задачe числа и всe относящиеся к даиным или к неизвeстным величинам условия;

3) послe такого примeнения всeх условий разыскать между составленными или просто записанными выраженияии два таких, которые в силу одного из данных условий должны быть равны между собою, и соeдинить эти выражения знаком равенства.

Примeним этот принцип к рeшению двух задач:

Задача 1 я. Число монет в одном кошелькe вдвое меньше, чeм в другом. Если выложить из первого шeсть монeт, а во второй прибавить восемь монет, то число монeт в первом окажется в семь раз менee, чeм во втором. Узнать, сколько монет в каждом кошелькe?

В этой задачe указаны нeсколько извeстных и нeсколько неизвeстных величин. Примем за первое неизвeстное число монет пeрвого кошелька и.обозначим его через х. Затeм займемся обозначeниeм всeх величин, к которым относятся условия задачи.

Число монeт перваго кошелька есть х . Отношениe чисел монет во втором и первом кошельках 2 . Значит число монет второго кошeлька 2х.

Из пeрвого вынимают 6 монeт. Поэтому в первом кошeлькe остаeтся монeт х -6 .

Во второй прибавляют 8 монет. Следовательно, во втором кошельке получится монет 2х +8 . Новое отношение между числами монет второго и первого кошелька есть . Оно также равно 7 . На этом основании составляем уравнение , решая которое, получим х= 10 , после чего нетрудно определить другие неизвестные, о которых мы здесь упоминали.

Если бы мы приняли за первое неизвестное число монет второго кошелька и обозначили бы его для отличия от предыдущаго обозначения через у , то, как легко убедиться, получилось бы другое уравнение, именно (у + 8 ):( у / 2 -6 )=7 , которое также разрешает задачу и дает ответ у =20 .

Можно было бы принять за первое неизвестиов число монвть, оказавшееся в первом кошельке после выкладки из него 6 монет; тогда, обозначив это неизвестное через z и идя тем же путем, каким мы шли при составлении первого уравнения, мы получили бы уравнение , откуда z = 4 .

Но можно было бы изменить также сам путь соотавления уравнения, напр., тем, что мы прежде приняли бы во внимание измененное отношение между числами монет, а составление уравнения основали бы на том, что известно о первоначальном отношении. В этом случае составление уравнения велось бы так:

Число монет первого кошелька после выкладки есть z . Выложено 6 монет. Значит первоначальное число монет первого кошелька z + 6. Измененное отношение между числами монет 7 . Поэтому измененное число монет второго кошелька 7z. Прибавлено было 8 монет. Следовательно, первоначальное число монет второго кошелька 7z. - 8 . Первоначальное отношение между числами монет есть Оно же равно 2 . На этом основании имеем уравнение , совместное с предыдущим, хотя и отличающееся от него по виду.

Если бы, идя этим вторым путем, мы приняли за первое неизвестное число монет второго кошелька после прибавления в него 8 монет, то, обозначив это неизвестное для отличия через и , получили бы уравнение (и -8 ):( и / 7 + 6 )=2 , откуда и =28 .

Эти разъяснения показывают, что, руководствуясь одним и тем же общим правилом для составления уравнений, мы все-таки получаем в каждой задаче разнообразные способы для достижения этой цели. Лучшим способом считается тот, который проще выражает условия задачи и быстрее ведет как к составлению, так и к решению уравнения. В данном случае первый и третий способы одинаково удобны для решения уравнения, но первый все-таки проще и потому лучше остальных.

Применяя указанное правило составления уравнений, нужно помнить, что во всякой правильно вираженной эадаче должно быть принято во внимание каждое данное число и каждое из выраженных условий.

Задача 2-я. Из города А выходит путешественник, проходящий в день по 20 верст. Через два дня навстречу ему выходит из города В другой путешественник, который проходит ежедневно по 30 верст. Расстояние между А иВ равно 190 верст. Спрашивается, когда и где встретятся оба путешественника?

1-й способ. Примем за первое неизвестное время движения первого путешественника от выхода из А до встречи, а за последнее условие то, что расстояние между А и В равно 190 верст. Тогда рассуждение будем вести так:

ІІоложим, что первый шел до встречи х дней. Ежедневно он проходил по 20 верст. Поэтому он прошел всего 20х верст.

Второй вышел позднее на 2 дня. Значит, он шел до встречи х -2 дня. Ежедневно он проходил по 30 верст. Следовательно, он прошел всего 30 (х -2 ) верст. Вместе оба путешественника прошли [20х + 30 (х -2 )] версть. Все расстояние между А и В равно 190 верст. На этом основании находим уравнение

20х + 30 (х -2 ) =190 ,

откуда х= 5 . Из этого видим, что первый путешественник шел 5 дней и прошел 100 верст, второй шел 3 дня и прошел 90 верст.

2-й способ. Примем за первое неизвестное расстояние, пройденное первым путешественником от выхода до встречи, и за последнее условие то, что второй путешественник вышел позднее первого на 2 дня. Тогда рассуждение поведется так:

Полагаем, что первый прошел до встречи у верст. Ежедневно он проходил по 20 верст. Поэтому он шел всего у / 20 дней.

Второй прошел всего (190 -у ) верст. Ежедневно он проходил по 30 верст. Значит он шел всего дней.

Разность между временами движения обоих есть и равна 2 . Следовательно, находим уравнение , откуда у =100 .

3-й способ. Первое неизвестное есть время движения второго путешественника от выхода из В до встречи, последнее условие то, что первый путашественник проходит ежедневно по 20 верст.

Положим, что второй идет до встречи z дней. Значит,первый пройдет (z +2 ) дня. Проходя ежедневно по 30 верcт, второй пройдет всeго 30z верст. Так как обоим нужно пройти 190 верст, то первому останется сделать (190 -30z ) верст. Для этого он должен делать ежедневно по верст. Так как это выражение равно 20 , то получается уравнение , откуда z = 3.

4-й способ. Первое неизвестное есть расстояние, пройденное вторым путешественником до встречи, последнее условие то, что второй проходит ежедневяо 10-ю верстами более первого.

Полагаем, что второй прошел до встречи и верст. Значит первому оставалось еще пройти (190 -и ) верст. Так как до выхода второго он уже прошел 40 верст, то после выхода второго ему оставалось еще пройти (150 -и ) верст. Разность расстояний, проходимых одновременно обоими, есть (2и -150 ) верст. Время их общего движения есть и / 30 дней.Следовательно, второй в день проходит больше первого на (2и -150 ) : и / 30 верст. Так как это выражение равно 10 , то получаетея уравнение (2и -150 ) : и / 30 =10 , которое дает и = 90 .

Предыдущие объяснения показывают, что разнообразие способов для составления уравнений в одной и той же задаче зависит как от порядка последовательно обозначаемых величин, так и от порядка последовательно принимаемых во внимание условий.

231. Два лица имеют вместе 38 рублей, причем у первого 6-ю рублями больше денег, чем у второго. Сколько денег у каждаого?

231. Два лица имеют вместе 114 рублей, причем у первого 18-ю рублями больше денег, чем у второго. Сколько денег у каждого?

232. В одном доме окон на 15 меньше, чем в другом, всего же в обоих домах 51 окно. Сколько окон в каждом?

232. В одном доме окон на 6 меньше, чем в другом; всего же в обоих домах 62 окна. Сколько окон в каждом?

233. В двух кошельках находится 81 рубль. В первом денег вдвое меньше, чем во втором. Сколько денег в каждом?

233. В двух кошельках находится 72 рубля. В первом денег в пять раз меньше, чем во втором. Сколько денег в каждом?

234. Отец старше сына втрое, а сумма лет обоих их равна 48 годам. Определить возраст обоих.

234. Отец старше сына вдвое, а сумма лет обоих равна 13 годам. Определить возраст обоих.

235. Сын моложе отда вчетверо, а разность их лет равна 27 годам. Сколько леть каждому?

235. Сын моложе отца впятеро, а разность их лет составляет 32 года. Сколько лет каждому?

236. В трех корзинах находится 47 яблок, причем в первой и во второй поровну, а в третьей на 2 яблока больше, чем в каждой из остальных. Сколько яблок в каждой корзине?

236. В трех корзинах находится 110 яблок, причем в первой и в третьей поровну, а во второй на 4 яблока меньше, чем в каждой из остальных. Сколько яблок в каждой корзине?

237. Три куска серебра весят вместе 48 фунтов. Первый тяжелее второго на 12 ф., а третий тяжелее первого на 9 фунтов. Сколько весит каждый кусок?

237. Три куска серебра весят вместе 33 ф.. Первый легче второго на 5 фунтов, а третий легче первого на 2 фунта. Сколько весит каждый кусок?

238. Сын моложе отца на 20 лет и старше дочери на 5 лет. Сумма лет всех троих равна 60 годам. Сколько лет каждому

238. Мать старше сына на 21 год и моложе отца на 7 лет. Сумма лет всех троих равна 64 годам. Сколько лет каждому?

239. На трех полках лежит всего 66 книг, причем на нижней втрое больше, а на средней вдвое болыьше, чем на верхней. Сколько книг на каждой полке?

239. На трех полках лежит всего 60 книг, причем на нижней в шесть раз больше, а на верхней в пять раз больше, чем на средней. Сколько книг на каждой полке?

240. Лес, сад и луг стоят вместе 10800 р.. Луг дороже сада в 2 раза, а лес дороже луга в три раза. Что стоит каждый из них отдельно?

240. Лес, сад и луг стоят вместе 17600 р.. Лес дороже сада в 3 раза, а луг дорожо леса в 4 раза. Что стоит каждый из них отдельно?

241. Разделить число 21 на две части так, чтобы кратное отношсние первой части ко второй равнялось дроби 3 / 4 .

241. Разделить число 48 на две части так, этобы кратное отношение второй части к первой равнялось дроби 5 / 3 .

242. Разделить число 88 на такие две части, чтобы частные от деления первой части на 5, а второй на 6 были равны.

242. Разделить число 55 на такие две части, чтобы частные от деления первой части на 7, а. второй на 4 были равны.

243. Сумма двух чисел 85, а разность их 15. Найти оба числа.

243. Сумма двух чисел 72, а разность их 8. Найти оба числа.

244. Разность двух чисел 8, а кратное отношение их равно дроби 3 / 2 .Найти эти числа.

244. Разность двух чисел 12, а кратное отношение их равно дроби 5 / 3 . Найти эти числа.

245. Разделить число 46 на две чаости так, чтобы разность частных от деления первой части на 3 и второй на 7 равнялась 2.

245. Разделить число 59 на две части так, чтобы разность частных от деления первой части на 3 и второй на 5 равнялась 1.

246. Разделить число 75 на две части так, чтобы большая часть превышала втрое разность между обеими частями.

246. Разделить число 56 на две части так, чтобы меньшая часть превышала втрое разность между обеими частями.

247. Сумма двух чисел 64. При делении большего числа на меньшее получается в частном 3 и в остатке 4. Найти эти числа.

247. Сумма двух чисел 45. При делении большего числа на меньшее получается в частном 5 и в остатке 3. Найти эти числа.

248. Разность двух чисел 35. При делении большего числа на меньшее получается в частном 4 и в остатке 2. Найти эти числа.

248. Разность двух чисел 23. При делении большего числа на меньшее получается в частном 2 и в остатке 11. Найти эти числа.

249. Одно из неизвестных двух чисел больше другого на 5. Если разделить меньшее число на 4, а большее на 3, то первое частное будет 4-мя меньше второго. Найти оба числа.

249. Одно из двух неизвестных чисел больше другого на 15. Если разделить большее число на 9, а меньшее на 2, то первое частное будеть 3-мя меньше второго. Найти оба числа.

250. Одно из двух неизвестных чисел меньше другого на 6. Если разделить большее число пополам, то полученное частное будет тремя единицами меньше другого числа. Найти оба числа.

250. Одно из двух неизвестных чисел меньше другого на 18. Если разделить большее число на три, то полученное частное будет двумя единицами больше другого числа. Найти оба числа.

251. В одном резервуаре вдвое больше воды, чем в другом; если же перелить из первого во второй 16 ведер, то в обоих окажется воды поровну. Сколько воды в каждом?

251. В одном резервуаре втрое больше воды, чем в другом; если же перелить из первого во второй 22 ведра, то в обоих окажется воды поровну, Сколько воды в каждом?

252. На рынке у двух торговок имеется всего 220 яиц; если бы вторая из них отдала первой 14 яиц, то число яиц у каждой из них оказалось бы одинаковым. Сколько яиц у каждой?

252. На рынке у двух торговок имеется всего 186 яиц; если бы вторая из них отдала первой 10 яиц, то число яиц у каждой из них оказалось бы одинаковым. Сколько яиц у каждой?

253. Некто имеет в правом кармане в 4 раза более рублей, чем в левом; если же он переложит из правого кармана в левый 6 р., то в правом окажется денег только в 3 раза более, чем в левом. Сколько денег в каждом кармане?

253. Некто ииеет в правом кармане в 3 раза более рублей, чем в левом; если же переложить из левого кармана в правый 5 рублей, то в правом окажется денег в пять раз более, чем в левом. Сколько денег в каждом кармане?

254. При расчете на фабрике двух рабочих первый из них получил за работу 12 рублями больше второго, и ему же после этого второй работник уплатил 2 руб. долгу. Оказалось, что первый понес домой денег втрое больше, чем второй. Сколько заработал каждый?

254. При расчете на фабрике двух рабочих первый из них получил за работу 20 рублями меньше второго, но при этом второй работник возвратил ему 2 руб. долгу. Оказалось, что первый понес домой денег вдвое меньше второго. Сколько заработал каждый?

255. У одного мальчика 30 копеек, у другого 11 коп.. Сколько раз им следуегь дать по одной копейке, чтобы у первого оказалось денег вдвое больше, чем у второго?

255. У одного мальчика 48 копеек, у другого 22 коп.. Сколько раз они должны истратить по одной копейке, чтобы у первого оказалось втрое больше денег, чем у второго?

256. Отцу 40 лет, а сыну 12 лет. Сколько лет тому назад отец был впятеро старше сына?

256. Отцу 49 лет, а сыну 11 лет. Через сколько лет отец будет втрое старше сына?

257. Один помещик имеет овец вчетверо больше, чем другой. Если бы оба прикупили по 9 овец, то у первого было бы овец втрое больше, чем у второго. Сколько овец у каждого?

257. Один помещнк имеет овец втрое меньше, чем другой. Если бы оба продали по 10 овец, то у первого оказалось бы овец впятеро меньше, чем у второго. Сколько овец у каждого?

258. Отец на 39 лет старше сына, а через 7 лет будет старше сына в 4 раза. Сколько лет тому и другому?

258. Отцу и сыну вместе 88 лет, а 8 лет тому назад отец был старше сына в 7 раз. Сколько лет тому и другому?

259. В одном резервуаре 48 ведер, а в другом 22 ведра воды. Из первого отлили воды вдвое больше, чем из второго, и тогда в первом осталось втрое больше воды, чем во втором. Сколько ведер вылито из каждого?

259. В одном резервуаре 42 ведра, а в другом 8 ведер воды. В первый прилито было воды втрое больше, чем во второй, и тогда оказалось в первом в четыре раза больше воды, чем во втором. Сколько ведер прилито в каждый?

260. Два лица, играя отдельно в карты, имели при начале игры- первый 72 рубля, второй 21 рубль. Первый проиграл втрое больше того, сколько второй выиграл. После игры оказалось у первого вдвое больше денег, чем у второго. Сколько выиграл второй и проиграл первый?

260. Два лица, играя отдельно в карты, имели при начале игры- первый 25 рублей, второй 12 рублей. Первый выиграл вдвое больше того, сколько второй проиграл. После игры оказалось у первого впятеро больше денег, чем у второго. Сколько проиграл второй и выиграл первый?

261. Разносчик продал в первый раз часть 2 / 7 числа бывших у него яблох, во второй раз р того же числа; тогда у него осталось всего 8 яблок. Сколько у него было яблок?

261. Разносчик продал в первый раз 1 / 9 числа бывших у него яблок, во второй раз 5 / 6 того же числа; тогда у него осталось всего 4 яблока. Сколько у него было яблок?

262. Из резервуара с водой отлита была сначала треть всего количества воды, затем 5 / 6 остатка и тогда осталось только 6 ведер. Сколько было воды в резервуаре?

262. Из резервуара с водой отлита была сначала часть 3 / 5 всего количества, затем 3 / 4 остатка и тогда осталось только 5 ведер. Сколько было воды в резервуаре?

263. В одном обществе было 40 человек мужчин, женщин и детей. Число женщиы составляло 3 / 5 числа мужчин, а число детей составляло 2 / 3 числа мужчин и женщин вместе. Сколько было мужчин, женщин и детей?

263. В одном обществе было 72 человека мужчин, женщин и детей. Число мужчин составляло 2 / 3 числа женщин, а число детей составляло 4 / 5 числа мужчин и женщин вместе. Сколько было мужчин, женщин и детей?

264. За 30 аршин сукна двух сортов заплачено всего 128 рублей; аршин первого сорта стоит 4 1 / 2 р., а аршин второго 4 р.. Сколько куплено аршивн того и другого сорта?

264. За 27 аршин сукна двух сортов заплачено всего 120 р.; аршин первого сорта стоит 5 руб.; аршин второго 3 р. 75 к.. Сколько куплено аршин того и другого серта?

265. Чайиый торговец продал 38 фунтов чаю двух сортов, ценою по 3 р. за фунт первого сорта и по 1 р. 60 к. за фунт второго сорта, и выручил при этом за весь первый сорт 22-мя рублями больше, чем за второй. Сколько продано чаю того и другого сорта?

265. Чайный торговец продал 110 фунгов чаю двух сортов, ценою по 4 1 / 2 р. за фунт первого сорта и по 2 р. 25 к. за фунт второго сорта, и выручил при этом за первый сорт 45-ю рублями меньше, чем за второй. Сколько продано чаю того и другого сорта?

266. Подрядчик нанял работника с условием платить ему 90 коп. за каждый рабочий день и вычитать с него 40 коп. за каждый нерабочий день. По прошествии 12 дней рабочий получил 6 р. 90 к.. Сколько дней он работал?

266. Подрядчик нанял работника с условием платить ему по 80 коп. за каждый рабочий день и вычитать с него 50 коп. за каждый нерабочий день. По прошествии 50 дней рабочий получил 21 р. 80 в.. Сколько дней он прогулял?

267. А и В играют на биллиарде с условием, что выигравший партию получаоет с проигравшаего 76 к.; после 20 партий оказалось, что В выиграл всего 4 р. 50 к.. Сколько партий он выиграл?

267 А и В играют на биллиарде с условием, что выигравший партию получаоет с проигравшаего 50 к.; после 12 партий оказалось, что А выиграл всего 2 р.. Сколько партий он проиграл?

268. Два курьера выехали одновременно из двух городов, находящихся на расстоянии 300 верст, и едут навстречу один другому. Первый проезжает в час 12 верст, второй 13 верст. Когда они встретятся?

268. Два курьера выехали одновременно из двух городов, находящихся на расстоянии 280 верст, и едут навстречу один другому. Первый проезжает в час 11 верст, второй 17 верст. Когда они встретятся?

269. С двух станций железной дороги, находящихся в расстоянии 77 верст, выходят одновременно два поезда и идут по одному направлению со скоростями 31 1 / 2 версть и 18 2 / 3 верст в час, причем первый идет за вторым. Когда он догонит?

269. С двух станций железной дороги, находящихся в расстоянии 38 верст, выходят одновременно два поезда и идут по одному направлению со скоростями 25 1 / 4 верст и 20 1 / 2 верст в час, причем первый идет за вторым. Когда он догонит?

270. Со станции в 12 ч. дня выходит пассажирский поезд, делающий по 32 в. в час. Через 45 минут с той же станции выходит курьерский поезд, делающий по 42 в. в час. В котором часу курьерский поезд догонит пассажирский?

270. Со станции в 9 часов утра выходит пассажирокий поезд, делающий по 28 в. в час. Через час с четвертью с той же станции выходит курьерский поезд, делающий по 40 в. в час. В котором часу курьерский поезд догонит пассажирский?

271. Какой капитал нужно отдать в рост по 6%, чтобы через 1 год 2 месяца получить прибыли 224 р.?

271. Какой капитал нужно отдать в рост по 8%, чтобы в 7 месяцев получить прибыли 182 р.?

272. По скольку процентов нужно отдать в рост капитал 4400 руб., чтобы через 1 год 5 месяцев получить прибыли 280 р. 50 к.?

272. По скольку процентов нужно отдать в рост капигал 1800 р., чтобы через 11 месяцев получить прибыли 93 р. 60 к.?

273. Купец, продав товар за 299 р., выручил 15% прибыли. Что стоит товар ему самому?

273. Купец, продав товар за 161 р., получил 7 1 / 2 % прибыли. Что стоит товар ему самому?

274. При продаже товара на сумму 429 р. получено убытку 2 1 / 2 %. Что стоит товар?

274. При продаже товара на сумму 366 р. получено убытку 8 1 / 2 % Что стоит товар?

275. По векселю за 10 месяцев до срока было уплачено 1120 р., при коммерческом учете по 8%. Найти валюту векселя.

275. По векселю за 1 год 3 месяца до срока было уплачено 839 р. 60 коп. при коммерческом учете по 7%. Найти валюту векселя.

276. Бассейн наполаяется одной трубой в 3 часа, другой в 5 часов. Во сколько времени наполнится он, если открыть одновременно обе трубы?

276. Басеейн наполняется одной трубой в 7 1 / 2 часов, другой в 5 часов. Во сколько времени наполнится он, если открыть одновременно обе трубы?

277. Бассейн наполняется одной трубой в 4 часа, а через другую может весь вытечь в 6 часов. Во сколько времени наполнится бассейн при одновременном действии обеих труб?

277. Бассейн наполняется одной трубой в 2 1 / 3 часа, а через другую может весь вытечь в 2 ч. 48 м.. Во сколко времени наполнится бассейн при одновременном действии обеих труб?

278. Два работника вместе кончают работу в 3 часа 36 мин.; один первый может ее исполнить в 6 часов. Во сколько времени сделает ту же работу второй?

278. Два работника вместе кончают работу в 12 часов; один первый может ее исполнить в 20 часов. Во сколько времена сделает ту же работу второй?

279. В бассейн проведены три трубы; через первые две вода вливается, через третью вытекает. Через первую трубу бассейн может наполниться в 3 часа, через вторую в 2 часа, а через третью вся вода может вытечь из бассейна в 6 часов. Во сколько времени бассейн наполнится, если открыть все три трубы?

279. В бассейн проведены три трубы; через первые две вода вливается, через третью вытекает. Через первую трубу бассейн может наполниться в 2 часа, через вторую в 5 часов, а чероез третью вся вода может вытечь из бассейна в 10 часов. Во сколько времени бассейн наполнится, если открыть все три трубы?

280. Из трех труб, проведенных в бассейн, первая наполняет его в 5 часов, вторая наполняет в 15 часов, а через третью весь бассейн вытекает в 3 часа. Во сколько времени полный бассейн вытечет при одновременном действин всех труб?

280. Из трех труб, проведенных в бассейн, первая наполняет его в 6 часов, вторая наполняет в 18 часов, а через третью весь бассейн вытекает в 3 часа. Во сколько времени полный бассейн вытечет при одновременном действии всех труб?

281. ІІоезд железной дороги идет из А в В со средней скоростью 30 верст в час, затем возвращается из В в А со скоростью 28 верст в час. Весь проезд туда и обратно он делает в 14 1 / 2 часов. Сколько верст от А до В ?

281. ІІоезд железной дороги идет из А в В со средней скоростью 24 версты в час, затем возвращается из В в А со скоростью 30 верст в час. Весь проезд туда и обратно он делает в 11 1 / 4 часов. Сколько верст от А до В ?

282. Из А в В вышел поезд, проходящий в час 20 верст. Черезь 8 часов выходит поезд из В в А , проходящий 30 в. в час. Расстояние АВ равно 350 в.. На каком расстоянии от А поезда встретятся?

282. Из А в В вышел поезд, проходящий в час 24 версты. Через 5 часов выходит поезд из В в А , проходящий 28 в. в час. Расстояние АВ равно 380 в., На каком расстоянии от В поезда встретятся?

283. Сумма трех чисел равна 70. Второе число при делении на первое дает в частном 2 и в остатке 1, третье при делении на второе дает в частном 3 и в остатке 3. Найти эти числа.

283. Сумма трех чисел равна 60. Второе число при делении на первое дает в частном 3 и в остатке 2, третье при делении на второе дает в частном 2 и в остатке 4. Найти числа.

284. Найти чиесло, которое при делении на 5 дает в остатке 2, а при деления на 8 дает в остатке 5, зная притом, что первое частное тремя больше второго.

284. Найти число, которое при делении на 7 дает в остатке 2, а при делении на 9 дает в остатке 4, зная притом. что первое частное двумя больше второго.

285. Некто, желая раздать имевшиеся при нем деньги нищим, рассчитал, что если каждому дать по 15 копеек, то у него не хватит 10 коп., а если каждому дать по 13 коп., то останется 6 к. лишних. Сколько было нищих и сколько денег?

285. Некто, желая раздать имевшиеся при нем деньги нищим, рассчитал, что если каждому дать по 8 коп., то останется 4 коп. лишних, а если каждому дать по 9 коп., то не хватит 2 коп.. Сколько было нищих и сколько денег?

286. Инженер размещает телеграфные столбы на некотором расстоянии. Если бы он поставил их на расстоянии 25 сажен один от другого, то надо было бы сделать еще 150 столбов, а если бы он увеличил расстояния между столбами на 5 сажен, то 70 столбов оказались бы лншними. Как велико расстояние и сколько изготовлено столбов?

286. Инженер размещаот телеграфные столбы на некотором расстоянии. Если бы он поставил их на расстоянии 30 сажен один от другого, то у него осталось бы лишних 100 столбов, а если бы он уменьшил расстояние столбов на 4 сажени, то надо было бы сделать еще 180 столбов. Как велико расстояние и сколько изготовлено столбов?

287. Некто при найме слуги обещал ему за год службы уплатить деньгамжи 144 руб. и дать одежду. Слуга расчелся через 7 месяцев а получил в уплату одежду и 54 рубля. Что стоила одежда?

287. Некто при найме слуги обещал ему за 7 месяцов службы уплатить деньгами 75 рублей и дать одежду. Слуга расчелся через 5 месяцев и получил в уплату одежду и 45 рублей. Что стоит одежда?

288. Заплачено за 46 пудов сахару на 195 руб. более, чем за 73 фунта чаю; 9 пудов сахару стоят на 30 рублей дешевле, чем 37 фунтов чаю. Что стоит фунт чаю и пуд сахару?

288. Заплачено за 21 фунт чаю на 238 рублей менее, чем за 40 пудов сахару; 15 фунтов чаю стоят на 2 руб. дороже, чем 4 пуда сахару. Что стоит фунт чаю и пуд сахару?

289. Помещик нанял двух крестьян за одинаковую поденную плату. Одному из них за 40 дней он отдал 7 р. 50 к. деньгами и 3 1 / 2 четверти овса, другому за 24 дня 4 руб. 80 к. деньгами и 2 четверти овса. Что стоит четверть овса?

289. Помещик нанял двух крестьян за одинаковую поденную плату. Одному из них за 56 дней он отдал 14 р. деньгами и 8 четвертей овса, другому за 88 дней 13 р. 50 к. деньгами и 15 четвертей овса. Что стоить четверть овса?

290. Заплачено за 25 аршин сукна и 21 арш. бархата 247 рублей. Известно, что 10 арш. бархата стоят 18-ю рублями дороже 13 аршин сукна. Что стоит аршин того и другого?

290. Заплачено за 15 аршин бархата и 52 арш. сукна 276 рублей. Известно, что 2 арш. бархата стоят 17-ю рублями дошевле 11 арш. сукна. Что стоит аршин того и другого?

291. Сумма цифр некоторого двузначного числа равна 12. Если от искомого числа отнят 18, то получится число, обозначенное теми же цифрами, но написанными в обратном порядке. Найти это число.

291. Разность цифр единиц и десятков некоторого двузначного числа равна 3. Если к искомому числу прибавить 27, то получится число, обозначенное теми же цифрами, но написанными в обратном порядке. Найти это число.

292. В некотором двузначном числе число десятков вдвое более числа единиц. Если цифры зтого числа переставим, то получим число, меньшее искомого на 36. Найти это число.

292. В некотором двузначном числе число десятков втрое менее числа единиц. Если цифры зтого числа переставим, то получим число, большее искомого на 36. Найти это число.

293. А играет в шашки с В и выигрывает у него из каждых четырех партий три, потом играет с С и у последнего выигрывает из каждых трех партий две. Всего А сыграл 21 партию и выиграл из них 15. Сколько партий сыграл он с В и с С ?

293. А играет в шашки с В и проигрывает ему из каждых восьми партий три, потом играет с С и проигрывает последному из каждых пяти партий две. В общем А сыграл 26 партий и проиграл из них 10. Сколько партий сыграл он с В и с С ?

294. Который теперь час, если 1 / 5 числа часов, прошедших от полудня, равна 1 / 3 числа часов, оставшихся до полуночи?

294. Который теперь час, если 1 / 11 числа часов, прошедших от полудня, равна 1 / 13 числа часов, оставшихся до полуночи?

295. Найти вес рыбы, зная, что хвост ее весит 2 ф., голова весит столько, сколько весит хвост и половина туловища, а туловище весит столько, столько голова и хвост.

295. Найти вес рыбы, зная, что голова ее весит 7 ф., хвост весит столько, сколько весит голова и половина туловища, а туловище весит сколько, сколько хвост и голова.

296. Некоторая сумма должна быть разделена можду двумя лицами так, чтобы части первого и второго относились между собой, как числа 5 и 3, и чтобы часть первого была на 50 руб. более 5 / 9 всей суммы. Как велика часть каждого?

296. Некоторая сумма должна быть разделена между двумя лицами так, чтобы части перваого и второго относились между собою, как числа 7 и 4, и чтобы часть второго была на 21 руб. меньше 5 / 12 всей сумиы. Как велика часть каждого?

297. Товар продан с убытком за 420 руб.; если бы его продали за 570 р., то полученная прибыль была бы в 5 раз более понесенного убытка. Что стоит товар?

297. Товар продан с прибылью за 520 р.; если бы его продали за 320 р., то получился бы убыток, составляющий 3 / 7 вырученной прибыли. Что стоит товар?

298. Числа аршин ситцу, содержащихся в трех кусках, относятся как 2:3:5. Если отрезать от первого куска 4 аршина, от второго 6 арш. и от третьего 10 арш., то оставшееся количество всего ситца составит 5 / 6 прежнего количества. Сколько аршин в каждом куске?

298.Числа аршин ситцу, содержащихся в трех кусках, относятся как 3:5:8. Если отрезать от первого 10 аршин, от второго 20 арш. и от третьего 30 арш., то оставшееся количество всего ситца составит 5 / 8 прежнего количества. Сколько аршин в каждом куске?

299. Из резервуара вылита сначала половина всей бывшей в нем воды и полведра, потом половина остатка и полведра, наконец еще половина остатка и полведра; после этого в резервуаре осталось 6 ведер. Сколько было воды вначале?

299. Из резервуара вылита треть бывшей в нем воды и треть ведра, потом треть остатка и треть ведра, наконец еще треть остатка и треть ведра; после этого в резервуаре осталось 7 ведер Сколько было воды вначале?

300. Несколько лиц делят некоторую сумму следующим образом; первый получает 100 р. и пятую часть остатка, второй 200 рублей и пятую часть нового остатка, третий 300 рублей и пятую часть остатка и т. д.. Оказалось, что вся сумма разделена на равные части. Как велика эта сумма, сколько участников в дележе и сколько досталось каждому?

300. Несколько лиц делят некоторую сумму следующим образом: первый получает 50 рублей и шестую часть остатка, второй 100 рублей и шестую часть нового остатка, третий 150 рублей и шестую часть остатка и т. д.. Оказалось, что вся сумма разделена на равные части. Как велика эта сумма, сколько участников в дележе и сколько досталось каждому?

Нижеследующие задачи отличаются от предыдущих тем, что данные выражены неявно, именно буквами. Эти задачи принадлежат к таким же типам, как прежние. При решении их повторяются важнейшие из тех приемов, которые применялись раньше, но, вследствие неявного вида данных, рассуждения имеют более общий и вместе с тем более отвлеченный характер. В новых упражнениях нужно так же, как и в прежних, заботиться прежде всего о том, чтобы выразить через главное неизвестное и через данные обозначения все величины, о которых в задаче прямо говорится, или которые в ней подразумеваются, и при этом нужно последовательно принимать во внимание все обозначения, данные в задаче, и все условия, относящиеся к данным и к искомым, когда таким образом все условия будут употреблены в дело, то сама собой явится мысль о том, как составить требуемое уравнение.

301. Разность двух чисел s q . Найти оба числа.

301.Разность двух чисел d , кратное отношение большего к меньшему q . Найти оба числа.

302. Разделить число а на три части так, чтобы первая часть была больше второй на число т и меньше третьей в п раз.

302. Разделить число а на три части так, чтобы первая часть была меньше второй на число т и больше третьей в п раз.

303. Одно число в а раз меньше другого. Если прибавить к,первому числу т , а ко второму п , то первая сумма будет в b раз меньше второй. Найти эти числа.

303. Одно число в а раз меньше другого. Если отнять от первого числа т , а от второго п , то первая разность будет в b раз больше второй. Найти эти числа.

304. Числитсль дроби меньше ее знаменателя на число а ; Если же от обоих членов дроби отнять по b т / п . Найти члены дроби.

304. Числитель дроби больше ее знаменателя на число а . Если же прибавить к обоим членам дроби по b , то получится дробь, равная дроби т / п . Найти члены дроби.

305. Разделить число а р раз больше второй и в q раз меньше третьей.

305. Разделить число а на такие три части, чтобы первая была. в р раз меньше второй и в q раз больше третьей.

306. Знаменатель дроби большое ее числителя в а раз. Если прибавить к числителю число b и вычесть из знаменателя число с , то получится дробь, равная дроби k / l . Найти члены дроби.

306. Знаменатель дроби меньшве ее числителя в а раз. Если вычесть из числителя число b и прибавнть к знаменателю число с , то подучится дробь, равная дроби k / l . Найти члены дроби.

307. Разделить число т на две части так, чтобы разность частных от деления первой части на а и второй на b раваялась бы r.

307. Разделить число т на две части так, чтобы сумма частных от деления первой части на а и второй на b равнялась бы s .

308. Работник за каждый рабочий день получает по а копеек, а за каждый нерабочий с него вычитают по b копеек. По прошествии п дней чистая выручка рабочего равна s рублям. Сколько было рабочих дней и сколько нерабочих?

308. Работник за каждый рабочий день получает по а копеек а за каждый нерабочий с него вычитают по b копеек. По прошествии п дней работник должен сам уплатить 5 рублей, Сколько было рабочих дней и сколько нерабочих?

309. Разность двух чисел d . При делении уменьшаемого на вычитаемое получается частное q и остаток, равный половине разности. Найти эти числа

309. Разность двух чисел d . При делении уменьшаемого на вычитаемое получается остаток r и частное, равное половине разности. Найти эти числа.

310. За несколько аршин сукна. заплачено а рублей; если бы купили сукна более на с b

310. За несколько аршин сукна заплачено а рублей; если бы купили сукна менее на с аршин, то нужно было бы заплатить b рублей. Сколько аршин куплено?

311. Какое число, будучи умножено на a , увеличится на число т ?

311. Какое число, будучи разделено на а , уменьшится на число т ?

312. При продаже дома за m рублей получено р процентов убытку. Что стоил он самому продавцу?

312. При продаже дома за т рублсй получено р процентов прибыли. Что стоил он самому продавцу?

313. Два курьера выезжают одновременно из двух мест А и В и едут по одному направлению от А к В и далее. ІІервый проезжает в час а верст, второй b верст. Расстояние АВ равно d верст. Когда и на каком расстоянии от А первый курьер догонит второго?

313. Два курьера выезжают одновременно из двух мест А и В и едут навстречу один другому. Первый проезжает в час а верст, второй b верст. Расстояние АВ равно d верст. Когда. и на каком расстоянии от А оба курьера встретятся?

314. Переднее колесо экипажа имеет окружность в а футов, окружность заднего b футов. Какое расстояние должен пройти экипаж, чтобы переднее колесо сделало на п оборотов большо заднего?

314. Переднее колесо экипажа имеет окружность на а футов меньшую, чем заднее. Какое расстояние должен пройти экипаж, чтобы переднее колесо сделало т , а заднее п оборотов?

315. В бассейн проведены две трубы, которые обе наполняют его, первая при отдельном действии в а часов, вторая также при отдельном действии в b часов. Во сколько времени наполнится бассевйн при одновременном действии обеих труб?

315. В бассейн проведены две трубы, из которых первая при отдельном действии наполняет его в а часов, а вторая также при отдельном действии выливает весь бассейн в b часов. Во сколько времени наполнится бассейн при одновременном дeйствии обeих труб?

316. Окружность заднего колеса экипажа в а раз большe окружности переднего колеса. Экипаж проeхал т футов, и при этом переднеe колесо сдeлало к оборотами большe заднего. Опредeлить окружности обоих колес и числа оборотов.

316. Окружность переднего колеса на а футов меньше окружности заднего. Экипаж проeхал т футов, и при этом заднеe колесо сдeлало в к раз меньше оборотов, чeм переднеe. Опрeдeлить окружности обоих колес и числа оборотов.

317. Народонаселение одного города увеличивается ежегодно на р % сравнительно с народонаселением предыдущего года. В настоящео время в городe т

317. Народонаселениe одного города уменьшается ежегодно на р % сравнительно с народонаселением предыдущего года. В настоящсе время в городe т жителей. Сколько было жителей 3 года назад?

318. Двоe рабочих, работая одновременно, кончают работу в а часов. Один первый сдeлает ту же работу в b , раз скорee, чeм один второй. Во сколько времени каждый из рабочих кончит работу?

318. Двоe рабочих, работая одновременно, кончают работу в а часов. Один пeрвый сдeлает ту жое работу в b , раз медлeннee, чeм один второй. Во сколько времени каждый из рабочих кончает работу?

319. Лодочник, гребя по течению рeки, проплывает п сажeн в t часов; грeбя жe против течения, он употребляeт на и часов болee, чтобы проплыть то жe расстояние. Опредeлить часовую скорость течения.

319. Лодочник, гребя против тeчения, проплывает п сажен в t часов; гребя жe по течению, он употребляет на и часов мeнee, чтобы проплыть то жe расстояниe. Опредeлить часовую скорость течeния.

320. Тeло А движeтся со скоростью v мeтров в секунду. С какой скоростью должно было двигаться другое тeло В , вышeдшеe из того жe мeста t сeкундами раньшe, если оно было настигнуто тeлом А через и секунд послe начала движения этого тeла?

320. Тeло A движeтся со скоростью v мeтров в секунду. С какой скоростью должно двигаться другоe тeло В , выходящеe из того же мeста и секундами позже, если оно догоняeт тeло А через и секукнд послe начала своeго движения?

321. Из двух сортов товару, цeною в а рублей и в b рублей за фунт, составлено d т рублей за фунт получено s рублей убытку. Сколько фунтов того и другого сорта пошло на составлениe смeси?

321. Из двух сортов товару, цeною в а рублей и в b рублей за фунт, составлено d фунтов смeси. При продажe этой смeси по т рублей за фунт получено s рублей прибыли. Сколько фунтов того и другого сорта пошло вна составлениe смeеси?

322. Б бассейн, вмeщающий т ведeр, проведены двe трубы. Первая вливает в бассейн а ведер в час. Вторая выливает весь бассейн в b часов. Во сколко часов наполнится бассейн при одновременном дeйствии обeих труб?

322. В бассейн, вмeщающий т ведер, проведены двe трубы. Первая наполняет весь бассейн в а часов. Вторая в час выливает из бассейна b ведер. Во сколько часов наполнится бассeйн при одновременном дeйствии обeих труб?

323. Раздeлить число а на три части так, чтобы первая относилась ко второй, как т: п , а вторая к третьeй, как р: q.

323. Раздeлить число а на три части так, чтобы вторая относилась к первой, как т: п , а третья ко второй, как р: q.

324. Из двух мeст А и В п сажен, плывут навстрeчу друг другу двe лодки, управляемые гребцами с одинаковой силой. ІІервая, плывущая по тeчeнию, проходит всe расстояние АВ в t часов; вторая, плывущая против течeния, употребляет на то жe раcстояниe большe времени на и часов. Опрeдeлить часовую скорость течения.

324. Из двух мeст А и В на рeкe, отстоящих одно от другого на п сажен, плывут навстрeчу друг другу двe лодки, управляемые гребцами с одинаковой силой. Пeрвая, плывущая против тeчеяия, проходит всe расстояние АВ в t часов; вторая, плывущая по течению, употребляет на то же расстояниe меньше врeмени на и часов. Опредeлить часовую скорость течения.

325. Опредeлить капиталы трех лиц, зная, что первый со вторым имeют вмeстe т рублей, второй с третьим п рублей, и что капитал пeрвого в р раз мeньшe капитала третьего.

325. Опрeдeлить капиталы трех лиц, зная, что первый с третьим имeют вмeстe т рублей, второй с третьим п рублей, и что капитал первого в р раз больше капитала второго.

326. Два тeла движутся навстрeчу одно другому из двух мeст, находящихся в расстоянии d метров. Первоe движется со скоростью v метров в секунду. С какой скоростью должно двигаться второе тeло, eсли оно вышло на h сeкунд позднee первого и должно идти до встрeчи всeго п секунд?

326. Два тeла движутся навстрeчу одно другому из двух мeст, находящихся в расстоянии d метров. Первое движется со скоростью v метров в секунду. С какой скоростью должно двигаться второе тeло, если оно вышло на h секунд раньше первого и должно идти до встрeчи всего п секунд?

327. Вексель, учтенный коммерчески по р % за п лeт до срока, дает учет больший математического, сдeланный также по р % и за п лeт, на а рублей. Найти валюту всксeля.

327. Вексeль, учтенный коммерчески по р % за п лeт, стоит на т рублей дешевле, чeм при учетe матeматическом, сдeланном такжe по р % и за п лeт. На какую сумму дан вексель?

328. Два курьeра выeзжают из мeст А и B , находящихся в расстоянии d вeрст, и eдут навстрeчу, проeзжая в час- первый u версгь и второй v верст; выeзд первoго из А состоялся на h В . Опредeлить, когда и гдe встрeтятся курьеры?

328. Два курьера выeзжают из мeст А и B находящихся в расстоянии d вeрст, и eдут оба в одном и том же направлении, проeзжая в час-первый и верст и второй v верст; выeзд пeрваго из А состоялся на h часов раньше выeзда второго из B . Опредeлить, когда и гдe первый курьер догонит второго?

329. Раздeлить число а на такия три части, что если к пeрвой приложить т , вторую сначала уменьшииь на m , а затeм умножить на п , и третью раздeлить на п , то полученные результаты окажутся равными.

329. Раздeлить число а на такия три части, что если первую умоньшить на т , вторую сначала увеличить на т , потом умножить на п , и третью раздeлить на п , то получатся равныe результаты.

330. В бассейн проведены три трубы А, В и С . Через А и С вода вливается, через В А и В бассейн наполняется в т часов, при дeйствии А и C в п часов, при дeйствии В и С в р часов. Во сколько временни наполнится бассейн при одновременном дeйствии всeх трех труб?

330. В бассейн проведены три трубы А, В и С . Через А вода вливается, через В и С вытекает. При совмeстном дeйствии труб А и В бассейн наполняется в т часов, при дeйствии А и С в п часов, трубы В и С выливают весь бассейн в р часов. Во сколько времени весь бассейн вытечeт при одновременном дeйствии всeх трех труб?

Составить уравнение - значит выразить в математической форме связь между данными (известными) задачи и искомыми (неизвестными) ее величинами. Иногда эта связь, настолько явно содержится в формулировке задачи, что составление уравнения есть просто дословный пересказ задачи, на языке математических знаков.

Пример 1. Петров получил за работу на 160 руб. больше, чем половина суммы, которую получил Иванов. Вместе они получили 1120 руб. Сколько получили за работу Петров и Иванов? Обозначим через х заработок Иванова. Половина его заработка есть 0,5x; месячной заработок Петрова 0,5x + 160 вместе они зарабатывают 1120 руб.; математическая запись последней фразы будет

(0,5x + 160) + x = 1120.

Уравнение составлено. Решая его по раз установленным правилам, находим, заработок Иванова х = 640 руб.; заработок же Петрова 0,5x + 160=480 (руб.) .

Чаше, однако, случается, что связь между данными и искомыми величинами не указывается в задаче прямо; ее нужно установить, исходя из условий задачи. В практических задачах так и бывает почти всегда. Только что приведенный пример носит надуманный характер; в жизни почти никогда подобных задач не встречается.

Для составления уравнения поэтому нельзя дать вполне исчерпывающих указаний. Однако на первых порах полезно руководствоваться следующим. Примем за значение искомой величины (или нескольких величин) какое-нибудь наугад взятое число (или несколько чисел) и поставим себе задачу проверить, угадали ли мы правильное решение задачи или нет. Если мы сумели провести эту проверку и обнаружить либо то, что догадка наша верна, либо то, что она неверна (скорее всего случится, конечно, второе) , то мы немедленно можем составить нужное уравнение (или несколько уравнений) . Именно, запишем те самые действия, которые мы производили для проверки, только вместо наугад взятого числа введем буквенной знак неизвестной величины. Мы получим требуемое уравнение.

Пример 2. Кусок сплава меди и цинка объемом в 1 дм3 весит 8,14 кг. Сколько меди содержится в сплаве? (уд. вес меди 8,9 кг/дм3; цинка - 7,0 кг/дм3) .

Возьмем наугад число, выражающее искомый объем меди, например 0,3 дм3. Проверим, удачно ли мы взяли это число. Так как 1 кг/дм3 меди весит 8,9 кг, то 0,3 дм3 весят 8,9 * 0,3 = 2,67 (кг) . Объем цинка в сплаве есть 1 - 0,3 = 0,7 (дм3) . Вес его 7,0 0,7 = 4,9 (кг) . Общий вес цинка и меди 2,67 + + 4,9 = 7,57 (кг) . Между тем вес нашего куска, по условию задачи, 8,14 кг. Догадка наша несостоятельна. Но зато мы немедленно получим уравнение решение которого даст правильный ответ. Вместо наугад взятого числа 0,3 дм3 обозначим объем меди (в дм3) через х. Вместо произведения 8,9 0,3 = 2,67 берем произведшие 8,9 x. Это - вес меди в сплаве. Вместо 1 - 0,3 = 0,7 берем 1 - х; это - объем цинка. Вместо 7,0 0,7 = 4,9 берем 7,0 (1 - x) ; это - вес цинка. Вместо 2,67+4,9 берем 8,9 х + 7,0 (1 - х) ; это - общий вес цинка и меди. По условию он равен 8,14 кг; значит, 8,9 х + 7,0 (1 - x) = 8,14.

Решение этого уравнения дает x = 0,6. Проверку наугад взятого решения можно делать различными способами; соответственно этому можно получить для одной и той же задачи различные виды уравнения; все они, однако, дадут для искомой величины одно и, то же решение, такие уравнения называются равносильными друг другу.

Разумеется, после получения навыков в составлении уравнений нет нужды производить проверку наугад взятого числа: можно для значения искомой величины брать не число, а какую-нибудь букву (х, у и т. д.) и поступать так, как если бы эта буква (неизвестное) была тем числом, проверить которое мы собираемся.

Решение задачи обычно сводится к тому, чтобы путем логических рассуждений и вычислений найти значение какой-нибудь величины. Например, найти скорость, время, расстояние, массу какого-нибудь предмета или количество чего-то.

Такую задачу можно решить с помощью уравнения. Для этого искомое значение обозначают через переменную, затем путем логических рассуждений составляют и решают уравнение. Решив уравнение, производят проверку на то, удовлетворяет ли решение уравнения условиям задачи.

Содержание урока

Запись выражений, содержащих неизвестное

Решение задачи сопровождается составлением уравнения к этой задаче. На начальном этапе изучения задач желательно научиться составлять буквенные выражения, описывающие ту или иную жизненную ситуацию. Этот этап не является сложным и его можно изучать в процессе решения самой задачи.

Рассмотрим несколько ситуаций, которые можно записать с помощью математического выражения.

Задача 1 . Возраст отца x лет. Мама на два года младше. Сын младше отца в 3 раза. Запишите возраст каждого с помощью выражений.

Решение:

Задача 2 . Возраст отца x лет, мама на 2 года младше отца. Сын младше отца в 3 раза, дочь младше матери в 3 раза. Запишите возраст каждого с помощью выражений.

Решение:

Задача 3 . Возраст отца x лет, мама на 3 года младше отца. Сын младше отца в 3 раза, дочь младше матери в 3 раза. Сколько лет каждому, если общий возраст отца, мамы, сына и дочери составляет 92 года?

Решение:

В данной задаче помимо записи выражений, необходимо вычислить возраст каждого члена семьи.

Сначала запишем возраст каждого члена семьи с помощью выражений. За переменную x примем возраст отца, и далее пользуясь этой переменной составим остальные выражения:

Теперь определим возраст каждого члена семьи. Для этого нам нужно составить и решить уравнение. Все компоненты уравнения у нас уже готовы. Осталось только собрать их воедино.

Общий возраст в 92 года получился путем сложения возрастов папы, мамы, сына и дочери:

Для каждого возраста мы составили математическое выражение. Эти выражения и будут компонентами нашего уравнения. Давайте соберем наше уравнение согласно данной схеме и таблице, которая была приведена выше. То есть слова папа, мама, сын, дочь заменим на соответствующее им в таблице выражение:

Выражение, отвечающее за возраст мамы x − 3, для наглядности было взято в скобки.

Теперь решим получившееся уравнение. Для начала можно раскрыть скобки там, где это можно:

Чтобы освободить уравнение от дробей, умножим обе части на 3

Решим получившееся уравнение, пользуясь известными тождественными преобразованиями:

Мы нашли значение переменной x . Эта переменная отвечала за возраст отца. Значит возраст отца составляет 36 лет.

Зная возраст отца, можно вычислить возрасты остальных членов семьи. Для этого нужно подставить значение переменной x в те выражения, которые отвечают за возраст конкретного члена семьи.

В задаче было сказано, что мама на 3 года младше отца. Ее возраст мы обозначили через выражение x−3. Значение переменной x теперь известно, и чтобы вычислить возраст мамы, нужно в выражении x − 3 вместо x подставить найденное значение 36

x − 3 = 36 − 3 = 33 года маме.

Аналогично определяется возраст остальных членов семьи:

Проверка :

Задача 4 . Килограмм яблок стоит x рублей. Запишите выражение, вычисляющее сколько килограмм яблок можно купить на 300 рублей.

Решение

Если килограмм яблок стоит x рублей, то на 300 рублей можно купить килограмм яблок.

Пример . Килограмм яблок стоит 50 рублей. Тогда на 300 рублей можно купить , то есть 6 килограмм яблок.

Задача 5 . На x рублей было куплено 5 кг яблок. Запишите выражение, вычисляющее сколько рублей стоит один килограмм яблок.

Решение

Если за 5 кг яблок было уплачено x рублей, то один килограмм будет стоит рублей

Пример . За 300 рублей было куплено 5 кг яблок. Тогда один килограмм яблок будет стоит , то есть 60 рублей.

Задача 6 . Том, Джон и Лео на перемене пошли в столовую и купили по бутерброду и по кружке кофе. Бутерброд стоит x рублей, а кружка кофе — 15 рублей. Определите стоимость бутерброда, если известно, что за всё было уплачено 120 рублей?

Решение

Конечно, данная задача проста как три копейки и ее можно решить не прибегая к уравнению. Для этого из 120 рублей нужно вычесть стоимость трех кружек кофе (15 × 3) , и полученный результат разделить на 3

Но наша цель — составить уравнение к задаче и решить это уравнение. Итак, стоимость бутерброда x рублей. Куплено их всего три. Значит увеличив стоимость в три раза, мы получим выражение описывающее сколько рублей было уплачено за три бутерброда

3x — стоимость трех бутербродов

А стоимость трех кружек кофе можно записать как 15 × 3 . 15 это стоимость одной кружки кофе, а 3 множитель (Том, Джон и Лео), увеличивающий эту стоимость в три раза.

По условию задачи за все уплачено 120 рублей. У нас уже появляется примерная схема, что нужно делать:

Выражения, описывающие стоимость трех бутербродов и трех кружек кофе, у нас уже готовы. Это выражения 3x и 15 × 3 . Пользуясь схемой составим уравнение и решим его:

Итак, стоимость одного бутерброда составляет 25 рублей.

Задача решается верно только в том случае, если уравнение к ней составлено правильно. В отличие от обычных уравнений, по которым мы учимся находить корни, уравнения для решения задач имеют своё конкретное применение. Каждый компонент такого уравнения может быть описан в словесной форме. Составляя уравнение, обязательно нужно понимать для чего мы включаем в его состав тот или иной компонент и зачем он нужен.

Также необходимо помнить, что уравнение это равенство, после решения которого левая часть должна будет равняться правой части. Составленное уравнение не должно противоречить этой идее.

Представим, что уравнение это весы с двумя чашами и экраном, показывающим состояние весов.

В данный момент экран показывает знак равенства. Понятно почему левая чаша равна правой чаше — на чашах ничего нет. Состояние весов и отсутствие на чашах чего-либо запишем с помощью следующего равенства:

0 = 0

Положим на левую чашу весов арбуз:

Левая чаша перевесила правую чашу и экран забил тревогу, показав знак не равно (≠). Этот знак говорит о том, что левая чаша не равна правой чаше.

Теперь попробуем решить задачу. Пусть требуется узнать сколько весит арбуз, который лежит на левой чаше. Но как это узнать? Ведь наши весы предназначены только для проверки равна ли левая чаша правой.

На помощь приходят уравнения. Вспомним, что уравнение по определению есть равенство , содержащее в себе переменную значение которой требуется найти. Весы в данном случае играют роль этого самого уравнения, а масса арбуза это переменная, значение которой нужно найти. Наша цель правильно составить это уравнение. Понимай, выровнять весы так, чтобы можно было вычислить массу арбуза.

Чтобы выровнять весы, на правую чашу можно положить какой-нибудь тяжелый предмет. Например, положим туда гирю массой 7 кг.

Теперь наоборот правая чаша перевесила левую. Экран по прежнему показывает, что чаши не равны.

Попробуем на левую чашу положить гирю массой 4 кг

Теперь весы выровнялись. На рисунке видно, что левая чаша на уровне правой чаши. А экран показывает знак равенства. Этот знак говорит о том, что левая чаша равна правой чаше.

Таким образом мы получили уравнение — равенство, содержащее неизвестное. Левая чаша — это левая часть уравнения, состоящая из компонентов 4 и переменной x (массы арбуза), а правая чаша — это правая часть уравнения, состоящая из компонента 7.

Ну и нетрудно догадаться, что корень уравнения 4 + x = 7 равен 3. Значит масса арбуза равна 3 кг.

Аналогично дела обстоят и с другими задачами. Чтобы найти какое-нибудь неизвестное значение, к левой или к правой части уравнения добавляют различные элементы: слагаемые, множители, выражения. В школьных задачах эти элементы бывают уже даны. Остается только правильно структурировать их и построить уравнение. Мы же в данном примере занимались подбором, пробуя гири разной массы, чтобы вычислить массу арбуза.

Естественно, те данные которые даны в задаче сначала нужно привести к виду, при котором их можно включить в уравнение. Поэтому, как говорят «хочешь не хочешь, а думать придётся» .

Рассмотрим следующую задачу. Возраст отца равен возрасту сына и дочери вместе. Сын вдвое старше дочери и на двадцать лет моложе отца. Сколько лет каждому?

Возраст дочери можно обозначить через x . Если сын вдвое старше дочери, то его возраст будет обозначаться как 2x . В условии задачи сказано, что вместе возраст дочери и сына равен возрасту отца. Значит возраст отца будет обозначаться суммой x + 2x

В выражении можно привести подобные слагаемые. Тогда возраст отца будет обозначаться как 3x

Теперь составим уравнение. Нам нужно получить равенство в котором можно найти неизвестное x . Воспользуемся весами. На левую чашу положим возраст отца (3x ) , а на правую чашу возраст сына (2x )

Понятно почему левая чаша перевесила правую и почему экран показывает знак (≠) . Ведь логично, что возраст отца больше возраста сына.

Но нам нужно уравнять весы, чтобы можно было вычислить неизвестное x . Для этого к правой чаше нужно прибавить какое-нибудь число. Какое именно число указано в задаче. В условии было сказано, что сын моложе отца на 20 лет. Значит 20 лет это то самое число, которое нужно положить на весы.

Весы выровнятся, если мы эти 20 лет добавим на правую чашу весов. Иными словами, вырастим сына до возраста отца

Теперь весы выровнялись. Получилось уравнение , которое решается легко:

x мы обозначили возраст дочери. Теперь мы нашли значение этой переменной. Дочери 20 лет.

Ну и наконец вычислим возраст отца. В задаче было сказано, что он равен сумме возрастов сына и дочери, то есть (20 + 40) лет.

Вернемся к середине задачи и обратим внимание на один момент. Когда мы положили на весы возраст отца и возраст сына, левая чаша перевесила правую

Но мы решили эту проблему, добавив на правую чашу еще 20 лет. В результате весы выровнялись и мы получили равенство

Но можно было не добавлять к правой чаше эти 20 лет, а вычесть их из левой. Мы получили бы равенство и в таком случае

В этот раз получается уравнение . Корень уравнения по прежнему равен 20

То есть уравнения и являются равносильными. А мы помним, что у равносильных уравнений корни совпадают. Если внимательно посмотреть на эти два уравнения, то можно увидеть что второе уравнение получено путем переноса числа 20 из правой части в левую с противоположным знаком. А это действие, как было указано в предыдущем уроке, не меняет корней уравнения.

Также нужно обратить внимание на то, что в начале решения задачи возрасты каждого члена семьи можно было обозначить через другие выражения.

Скажем возраст сына обозначить через x и поскольку он двое старше дочери, то возраст дочери обозначить через (понимай сделать её младше сына в два раза). А возраст отца поскольку он является суммой возрастов сына и дочери обозначить через выражение . Ну и напоследок для построения логически правильного уравнения, к возрасту сына нужно прибавить число 20, ведь отец старше на двадцать лет. В итоге получается совсем другое уравнение . Решим это уравнение

Как видно ответы к задаче не поменялись. Сыну по прежнему 40 лет. Дочери по прежнему лет, а отцу 40 + 20 лет.

Другими словами, задача может решаться различными методами. Поэтому не следует отчаиваться, что не получается решить ту или иную задачу. Но нужно иметь ввиду, что существует наиболее простые пути решения задачи. К центру города можно доехать различными маршрутами, но всегда существует наиболее удобный, быстрый и безопасный маршрут.

Примеры решения задач

Задача 1. В двух пачках всего 30 тетрадей. Если бы из первой пачки переложили во вторую 2 тетради, то в первой пачке стало бы вдвое больше тетрадей, чем во второй. Сколько тетрадей было в каждой пачке?

Решение

Обозначим через x количество тетрадей, которое было в первой пачке. Если всего тетрадей было 30, а переменная x это количество тетрадей из первой пачке, то количество тетрадей во второй пачке будет обозначаться через выражение 30 − x . То есть от общего количества тетрадей вычитаем количество тетрадей из первой пачки и тем самым получаем количество тетрадей из второй пачки.

и добавим эти две тетради во вторую пачку

Попробуем составить уравнение из имеющихся выражений. Положим на весы обе пачки тетрадей

Левая чаша тяжелее правой. Это потому, что в условии задачи сказано, что после того как из первой пачки взяли две тетради и положили их во вторую, количество тетрадей в первой пачке стало вдвое больше, чем во второй.

Чтобы выровнять весы и получить уравнение, увеличим правую часть вдвое. Для этого умножим её на 2

Получается уравнение . Решим данное уравнение:

Первую пачку мы обозначали через переменную x . Теперь мы нашли её значение. Переменная x равна 22. Значит в первой пачке было 22 тетради.

А вторую пачку мы обозначали через выражение 30 − x и поскольку значение переменой x теперь известно, то можно вычислить количество тетрадей во второй пачке. Оно равно 30 − 22 , то есть 8 шт .

Задача 2 . Два человека чистили картофель. Один очищал в минуту две картофелины, а второй — три картофелины. Вместе они очистили 400 шт. Сколько времени работал каждый, если второй проработал на 25 минут больше первого?

Решение

Обозначим через x время работы первого человека. Поскольку второй человек проработал на 25 минут больше первого, то его время будет обозначаться через выражение

Первый рабочий в минуту очищал 2 картофелины, и поскольку он работал x минут, то всего он очистил 2x картофелин.

Второй человек в минуту очищал три картофелины, и поскольку он работал минут, то всего он очистил картофелин.

Вместе они очистили 400 картофелин

Из имеющихся компонентов составим и решим уравнение. В левой части уравнения будут картофелины, очищенные каждым человеком, а в правой части их сумма:

В начале решения данной задачи через переменную x мы обозначили время работы первого человека. Теперь мы нашли значение этой переменной. Первый человек работал 65 минут.

А второй человек работал минут, и поскольку значение переменной x теперь известно, то можно вычислить время работы второго человека — оно равно 65 + 25 , то есть 90 мин .

Задача из Учебника по алгебре Андрея Петровича Киселева . Из сортов чая составлена смесь в 32 кг. Килограмм первого сорта стоит 8 руб., а второго сорта 6 руб. 50 коп. Сколько килограммов взято того и другого сорта, если килограмм смеси стоит (без прибыли и убытка) 7 руб. 10 коп.?

Решение

Обозначим через x массу чая первого сорта. Тогда масса чая второго сорта будет обозначаться через выражение 32 − x

Килограмм чая первого сорта стоит 8 руб. Если эти восемь рублей умножить на количество килограмм чая первого сорта, то можно будет узнать во сколько рублей обошлись x кг чая первого сорта.

А килограмм чая второго сорта стоит 6 руб. 50 коп. Если эти 6 руб. 50 коп. умножить на 32 − x , то можно узнать во сколько рублей обошлись 32 − x кг чая второго сорта.

В условии сказано, что килограмм смеси стоит 7 руб. 10 коп. Всего же было приготовлено 32 кг смеси. Умножим 7 руб. 10 коп. на 32 мы сможем узнать сколько стоит 32 кг смеси.

Выражения из которых мы будем составлять уравнение теперь принимают следующий вид:

Попробуем составить уравнение из имеющихся выражений. Положим на левую чашу весов стоимость смесей чая первого и второго сорта, а на правую чашу положим стоимость 32 кг смеси, то есть общую стоимость смеси, в составе которой оба сорта чая:

В начале решения данной задачи через переменную x мы обозначили массу чая первого сорта. Теперь мы нашли значение этой переменной. Переменная x равна 12,8. Значит для приготовления смеси было взято 12,8 кг чая первого сорта.

А через выражение 32 − x мы обозначили массу чая второго сорта и поскольку значение переменой x теперь известно, то можно вычислить массу чая второго сорта. Оно равно 32 − 12,8 то есть 19,2 . Значит для приготовления смеси было взято 19,2 кг чая второго сорта.

Задача 3 . Велосипедист проехал некоторое расстояние со скоростью 8 км/ч. Возвратиться он должен был другой дорогой, которая была на 3 км длиннее первой, и, хотя возвращаясь, ехал со скоростью 9 км/ч, он употребил времени на минут более. Как длинны были дороги?

Решение

Некоторые задачи могут затрагивать темы, которые человек возможно не изучал. Данная задача относится к такому кругу задач. В ней затрагиваются понятия расстояния, скорости и времени. Соответственно, чтобы решить подобную задачу, нужно иметь представление о тех вещах, о которых говорится в задаче. В нашем случае, надо знать что представляет собой расстояние, скорость и время.

В задаче нужно найти расстояния двух дорог. Мы должны составить уравнение, которое позволит вычислить эти расстояния.

Вспомним, как взаимосвязаны расстояние, скорость и время. Каждая из этих величин может быть описана с помощью буквенного уравнения:

Правую часть одного из этих уравнений мы будем использовать для составления своего уравнения. Чтобы узнать какую именно, нужно вернуться к тексту задачи и обратить внимание на следующий момент:

Следует обратить внимание на момент, где велосипедист на обратном пути употребил времени на минут более. Эта подсказка указывает нам, что можно воспользоваться уравнением , а именно его правой частью. Это позволит нам составить уравнение, которое содержит переменную S .

Итак, обозначим длину первой дороги через S . Этот путь велосипедист проехал со скоростью 8 км/ч . Время за которое он преодолел этот путь будет обозначаться выражением , поскольку время это отношение пройденного расстояния к скорости

Обратная дорога для велосипедиста была длиннее на 3 км . Поэтому её расстояние будет обозначаться через выражение S + 3 . Эту дорогу велосипедист проехал со скоростью 9 км/ч . А значит время за которое он преодолел этот путь будет обозначаться выражением .

Теперь составим уравнение из имеющихся выражений

Правая чаша тяжелее левой. Это потому, что в задаче сказано, что на обратную дорогу велосипедист затратил времени на больше.

Чтобы уравнять весы прибавим к левой части эти самые минут. Но сначала переведем минуты в часы, поскольку в задаче скорость измеряется в километрах в час, а не в метрах в минуту.

Чтобы минут перевести в часы, нужно разделить их на 60

Минут составляют часа. Прибавляем эти часа к левой части уравнения:

Получается уравнение . Решим данное уравнение. Чтобы избавиться от дробей, обе части части можно умножить на 72. Далее пользуясь известными тождественными преобразованиями, найдем значение переменной S

Через переменную S мы обозначали расстояние первой дороги. Теперь мы нашли значение этой переменной. Переменная S равна 15. Значит расстояние первой дороги составляет 15 км.

А расстояние второй дороги мы обозначили через выражение S + 3 , и поскольку значение переменной S теперь известно, то можно вычислить расстояние второй дороги. Это расстояние равно сумме 15 + 3 , то есть 18 км .

Задача 4 . По шоссе идут две машины с одной и той же скоростью. Если первая увеличит скорость на 10 км/ч, а вторая уменьшит скорость на 10 км/ч, то первая за 2 ч пройдет столько же, сколько вторая за 3 ч. С какой скоростью идут автомашины?

Решение

Обозначим через v скорость каждой машины. Далее в задаче приводятся подсказки: скорость первой машины увеличить на 10 км/ч, а скорость второй — уменьшить на 10 км/ч. Воспользуемся этой подсказкой

Далее говорится, что при таких скоростях (увеличенных и уменьшенных на 10 км/ч) первая машина пройдет за 2 часа столько же расстояния сколько вторая за 3 часа. Фразу «столько же» можно понимать как «расстояние, пройденное первой машиной, будет равно расстоянию, пройденному второй машиной» .

Расстояние как мы помним, определяется по формуле . Нас интересует правая часть этого буквенного уравнения — она позволит нам составить уравнение, содержащее переменную v .

Итак, при скорости v + 10 км/ч первая машина пройдет 2(v+10) км , а вторая пройдет 3(v − 10) км . При таком условии машины пройдут одинаковые расстояния, поэтому для получения уравнения достаточно соединить эти два выражения знаком равенства. Тогда получим уравнение . Решим его:

В условии задачи было сказано, что машины идут с одинаковой скоростью. Мы обозначили эту скорость через переменную v . Теперь мы нашли значение этой переменной. Переменная v равна 50. Значит скорость обеих машин составляла 50 км/ч.

Задача 5 . За 9 ч по течению реки теплоход проходит тот же путь, что за 11 ч против течения. Найдите собственную скорость теплохода, если скорость течения реки 2 км/ч.

Решение

Обозначим через v собственную скорость теплохода. Скорость течения реки равна 2 км/ч. По течению реки скорость теплохода будет составлять v + 2 км/ч , а против течения — (v − 2) км/ч .

В условии задачи сказано, что за 9 ч по течению реки теплоход проходит тот же путь, что за 11 ч против течения. Фразу «тот же путь» можно понимать как «расстояние, пройденное теплоходом по течению реки за 9 часов, равно расстоянию, пройденному теплоходом против течения реки за 11 часов» . То есть расстояния будут одинаковыми.

Расстояние определяется по формуле . Воспользуемся правой частью этого буквенного уравнения для составления своего уравнения.

Итак, за 9 часов по течению реки теплоход пройдет 9(v + 2) км , а за 11 часов против течения — 11(v − 2) км . Поскольку оба выражения описывают одно и то же расстояние, приравняем первое выражение ко второму. В результате получим уравнение . Решим его:

Значит собственная скорость теплохода составляет 20 км/ч.

При решении задач полезной привычкой является заранее определить на каком ищется для неё решение.

Допустим, что в задаче требовалось найти время, за которое пешеход преодолеет указанный путь. Мы обозначили время через переменную t , далее составили уравнение, содержащее эту переменную и нашли её значение.

Из практики мы знаем, что время движения объекта может принимать как целые значения, так и дробные, например 2 ч, 1,5 ч, 0,5 ч. Тогда можно сказать, что решение данной задачи ищется на множестве рациональных чисел Q , поскольку каждое из значений 2 ч, 1,5 ч, 0,5 ч может быть представлено в виде дроби.

Поэтому после того, как неизвестную величину обозначили через переменную, полезно указать к какому множеству эта величина принадлежит. В нашем примере время t принадлежит множеству рациональных чисел Q

t Q

Ещё можно ввести ограничение для переменной t , указав что она может принимать только положительные значения. Действительно, если объект затратил на путь определенное время, то это время не может быть отрицательным. Поэтому рядом с выражением t Q укажем, что её значение должно быть больше нуля:

t R , t > 0

Если решив уравнение, мы получим отрицательное значение для переменной t , то можно будет сделать вывод, что задача решена неправильно, поскольку это решение не будет удовлетворять условию t Q , t > 0 .

Ещё пример. Если бы мы решали задачу в которой требовалось найти количество человек для выполнения той или иной работы, то это количество мы обозначили бы через переменную x . В такой задаче решение искалось бы на множестве натуральных чисел

x N

Действительно, количество человек является целым числом, например 2 человека, 3 человека, 5 человек. Но никак не 1,5 (один целый человек и половина человека) или 2,3 (два целых человека и еще три десятых человека).

Здесь можно было бы указать, что количество человек должно быть больше нуля, но числа входящие во множество натуральных чисел N сами по себе являются положительными и большими нуля. В этом множестве нет отрицательных чисел и числа 0. Поэтому выражение x > 0 можно не писать.

Задача 6 . Для ремонта школы прибыла бригада в которой было в 2,5 раза больше маляров, чем плотников. Вскоре прораб включил в бригаду еще четырех маляров, а двух плотников перевел на другой объект. В результате маляров в бригаде оказалось в 4 раза больше чем плотников. Сколько маляров и сколько плотников было в бригаде первоначально

Решение

Обозначим через x плотников, прибывших на ремонт первоначально.

Количество плотников является целым числом, большим нуля. Поэтому укажем, что x принадлежит множество натуральных чисел

x N

Маляров было в 2,5 раза больше, чем плотников. Поэтому количество маляров будет обозначаться как 2,5x .

А количество маляров увеличим на 4

Теперь количество плотников и маляров будут обозначаться через следующие выражения:

Попробуем составить уравнение из имеющихся выражений:

Правая чаша больше, поскольку после включения в бригаду ещё четырёх маляров, и перемещения двух плотников на другой объект, количество маляров в бригаде оказалось в 4 раза больше чем плотников. Чтобы уравнять весы, нужно левую чашу увеличить в 4 раза:

Получили уравнение . Решим его:

Через переменную x было обозначено первоначальное количество плотников. Теперь мы нашли значение этой переменной. Переменная x равна 8. Значит 8 плотников было в бригаде первоначально.

А количество маляров было обозначено через выражение 2,5 x и поскольку значение переменной x теперь известно, то можно вычислить количество маляров — оно равно 2,5 × 8 , то есть 20 .

Возвращаемся к началу задачи и удостоверяемся, что соблюдается условие x N. Переменная x равна 8, а элементы множества натуральных чисел N это все числа, начинающиеся с 1, 2, 3 и так далее до бесконечности. В это же множество входит число 8, которое мы нашли.

8 N

Тоже самое можно сказать о количестве маляров. Число 20 принадлежит множеству натуральных чисел:

20 N

Для понимания сути задачи и правильного составления уравнения, вовсе необязательно использовать модель весов с чашами. Можно использовать и другие модели: отрезки, таблицы, схемы. Можно придумать свою модель, которая хорошо описывала бы суть задачи.

Задача 9 . Из бидона отлили 30% молока. В результате в нем осталось 14 л. Сколько литров молока было в бидоне первоначально?

Решение

Искомое значение это первоначальное число литров в бидоне. Изобразим число литров в виде линии и подпишем эту линию как X

Сказано, что из бидона отлили 30% молока. Выделим на рисунке приблизительно 30%

Процент по определению есть одна сотая часть чего-то. Если 30% молока отлили, то остальные 70% остались в бидоне. На эти 70% приходятся 14 литров, указанные в задаче. Выделим на рисунке оставшиеся 70%

Теперь можно составить уравнение. Вспомним, как находить процент от числа. Для этого общее количество чего-то делят на 100 и полученный результат умножают на искомое количество процентов. Замечаем, что 14 литров, составляющих 70% можно получить таким же образом: первоначальное число литров X разделить на 100 и полученный результат умножить на 70. Всё это приравнять к числу 14

Или получить более простое уравнение: 70% записать как 0,70, затем умножить на X и приравнять это выражение к 14

Значит первоначально в бидоне было 20 литров молока.

Задача 9 . Взяли два сплава золота и серебра. В одном количество этих металлов находится в отношении 1: 9, а в другом 2: 3. Сколько нужно взять каждого сплава, чтобы получить 15 кг нового сплава, в котором золото и серебро относилось бы как 1: 4?

Решение

Попробуем сначала узнать сколько золота и серебра будет содержáться в 15 кг нового сплава. В задаче сказано, что содержание этих металлов должно быть в отношении 1: 4, то есть на одну часть сплава должно приходиться золото, а на четыре части — серебро. Тогда всего частей в сплаве будет 1 + 4 = 5, а масса одной части будет 15: 5 = 3 кг.

Определим сколько золота будет содержáться в 15 кг сплава. Для этого 3 кг умножим на количество частей золота:

3 кг × 1 = 3 кг

Определим сколько серебра будет содержáться в 15 кг сплава:

3 кг × 4 = 12 кг

Значит сплав массой 15 кг будет содержать 3 кг золота и 12 кг серебра. Теперь вернёмся к исходным сплавам. Использовать нужно каждый из них. Обозначим через x массу первого сплава, а массу второго сплава можно обозначить через 15 − x

Выразим в процентах все отношения, которые даны в задаче и заполним ими следующую таблицу:

В первом сплаве золото и серебро находятся в отношении 1: 9. Тогда всего частей будет 1 + 9 = 10 . Из них золота будет , а серебра .

Перенесём эти данные в таблицу. 10% занесём в первую строку в графу «процент золота в сплаве» , 90% также занесём в первую строку графу «процент серебра в сплаве» , а в последнюю графу «масса сплава» занесём переменную x , поскольку так мы обозначили массу первого сплава:

Аналогично поступаем со вторым сплавом. Золото и серебро в нём находятся в отношении 2: 3. Тогда всего частей будет 2 + 3 = 5. Из них золота будет , а серебра .

Перенесём эти данные в таблицу. 40% занесем во вторую строку в графу «процент золота в сплаве» , 60% также занесём во вторую строку графу «процент серебра в сплаве» , а в последнюю графу «масса сплава» занесём выражение 15 − x , поскольку так мы обозначили массу второго сплава:

Заполним последнюю строку. Полученный сплав массой 15 кг будет содержать 3 кг золота, что составляет сплава, а серебра будет сплава. В последнюю графу записываем массу полученного сплава 15

Теперь по данной таблице можно составить уравнения. Вспоминаем . Если мы отдельно сложим золото обоих сплавов и приравняем эту сумму к массе золота полученного сплава, то сможем узнать чему равно значение x .

В первом сплаве золота было 0,10x , а во втором сплаве золота было 0,40(15 − x ) . Тогда в полученном сплаве масса золота будет суммой масс золота первого и второго сплавов и эта масса составляет 20% от нового сплава. А 20% от нового сплава это 3 кг золота, вычисленные нами ранее. В результате получаем уравнение 0,10x + 0.40(15 − x ) = 3 . Решим это уравнение:

Изначально через x мы обозначили массу первого сплава. Теперь мы нашли значение этой переменной. Переменная x равна 10. А массу второго сплава мы обозначили через 15 − x , и поскольку значение переменной x теперь известно, то можно вычислить массу второго сплава, она равна 15 − 10 = 5 кг .

Значит для получения нового сплава массой 15 кг в котором золото и серебро относились бы как 1: 4, нужно взять 10 кг первого и 5 кг второго сплава.

Уравнение можно было составить, воспользовавшись и вторым столбцом получившейся таблицы. Тогда мы получили бы уравнение 0,90x + 0.60(15 − x ) = 12. Корень этого уравнения тоже равен 10

Задача 10 . Имеется руда из двух пластов с содержанием меди в 6% и 11%. Сколько надо взять бедной руды, чтобы получить при смешивании с богатой 20 тонн с содержанием меди 8%?

Решение

Обозначим через x массу бедной руды. Поскольку нужно получить 20 тонн руды, то богатой руды будет взято 20 − x . Поскольку содержание меди в бедной руде составляет 6%, то в x тоннах руды будет содержáться 0,06x тонн меди. В богатой руде содержание меди составляет 11%, а в 20 − x тоннах богатой руды будет содержáться 0,11(20 − x ) тонн меди.

В получившихся 20 тоннах руды содержание меди должно составлять 8%. Значит в 20 тоннах руды меди будет содержáться 20 × 0,08 = 1,6 тонн.

Сложим выражения 0,06x и 0,11(20 − x ) и приравняем эту сумму к 1,6. Получим уравнение 0,06x + 0,11(20 − x ) = 1,6

Решим данное уравнение:

Значит для получения 20 тонн руды с содержанием меди 8%, нужно взять 12 тонн бедной руды. Богатой же будет взято 20 − 12 = 8 тонн.

Задача 11 . Увеличив среднюю скорость с 250 до 300 м/мин спортсменка стала пробегать дистанцию на 1 мин быстрее. Какова длина дистанции?

Решение

Длину дистанции (или расстояние дистанции) можно описать следующим буквенным уравнением:

Воспользуемся правой частью этого уравнения для составления своего уравнения. Изначально спортсменка пробегала дистанцию со скоростью 250 метров в минуту. При такой скорости длина дистанции будет описываться выражением 250t

Затем спортсменка увеличила свою скорость до 300 метров в минуту. При такой скорости длина дистанции будет описываться выражением 300t

Заметим, что длина дистанции это величина постоянная. От того, что спортсменка увеличит скорость или уменьшит её, длина дистанции останется неизменной.

Это позволяет нам приравнять выражение 250t к выражению 300t , поскольку оба выражения описывают длину одной и той же дистанции

250t = 300t

Но в задаче сказано, что при скорости 300 метров в минуту спортсменка стала пробегать дистанцию на 1 минуту быстрее. Другими словами, при скорости 300 метров в минуту, время движения уменьшится на единицу. Поэтому в уравнении 250t = 300t в правой части время нужно уменьшить на единицу:

При скорости 250 метров в минуту спортсменка пробегает дистанцию за 6 минут. Зная скорость и время, можно определить длину дистанции:

S = 250 × 6 = 1500 м

А при скорости 300 метров в минуту спортсменка пробегает дистанцию за t − 1 , то есть за 5 минут. Как было сказано ранее длина дистанции не меняется:

S = 300 × 5 = 1500 м

Задача 12 . Всадник догоняет пешехода, находящегося впереди него на 15 км. Через сколько часов всадник догонит пешехода, если каждый час первый проезжает по 10 км, а второй проходит только по 4 км?

Решение

Данная задача является . Её можно решить, определив скорость сближения и разделив изначальное расстояние между всадником и пешеходом на эту скорость.

Скорость сближения определяется вычитанием меньшей скорости из большей:

10 км/ч − 4 км/ч = 6 км/ч (скорость сближения)

С каждым часом расстояние в 15 километров будут сокращаться на 6 км. Чтобы узнать, когда оно сократится полностью (когда всадник догонит пешехода), нужно 15 разделить на 6

15: 6 = 2,5 ч

2,5 ч это два целых часа и половина часа. А половина часа это 30 минут. Значит всадник догонит пешехода через 2 часа 30 минут.

Решим эту задачу с помощью уравнения.

После этого вслед за ним в путь вышел всадник со скоростью 10 км/ч. А скорость пешехода составляет только 4 км/ч. Это значит, что всадник через некоторое время догонит пешехода. Это время нам нужно найти.

Когда всадник догонит пешехода это будет означать, что они вместе прошли одинаковое расстояние. Расстояние, пройденное всадником и пешеходом описывается следующим уравнением:

Воспользуемся правой частью этого уравнения для составления своего уравнения.

Расстояние, пройденное всадником, будет описываться выражением 10t . Поскольку пешеход вышел в путь раньше всадника и успел преодолеть 15 км, то расстояние пройденное им будет описываться выражением 4t + 15 .

На момент, когда всадник догонит пешехода, оба они пройдут одинаковое расстояние. Это позволяет нам приравнять расстояния, пройденные всадником и пешеходом:

Получилось простейшее уравнение. Решим его:

Задачи для самостоятельного решения

Задача 1. Из одного города в другой пассажирский поезд приезжает на 45 мин быстрее товарного. Вычисли расстояние между городами, если скорость пассажирского поезда 48 км/ч, а товарного 36 км/ч.

Решение

Скорости поездов в данной задаче измеряются в километрах в час. Поэтому 45 мин, указанные в задаче, переведем в часы. 45 мин это 0,75 ч

Обозначим время, за которое товарный поезд приезжает в город, через переменную t . Поскольку пассажирский поезд приезжает в этот город на 0,75 ч быстрее, то время его движения будет обозначаться через выражение t − 0,75

Пассажирский поезд преодолел 48(t − 0.75) км, а товарный 36t км. Поскольку речь идет об одном и том же расстоянии, приравняем первое выражение ко второму. В результате получим уравнение 48(t − 0.75) = 36t . Решим его:

Теперь вычислим расстояние между городами. Для этого скорость товарного поезда (36 км/ч) умножим на время его движения t. Значение переменной t теперь известно — оно равно трём часам

36 × 3 = 108 км

Для вычисления расстояния можно воспользоваться и скоростью пассажирского поезда. Но в этом случае значение переменной

Значение переменной t равно 1,2. Значит автомобили встретились через 1,2 часа.

Ответ: автомобили встретились через 1,2 часа.

Задача 3. В трех цехах завода всего 685 рабочих. Во втором цехе рабочих в три раза больше, чем в первом, а в третьем — на 15 рабочих меньше, чем во втором цехе. Сколько рабочих в каждом цехе?

Решение

Пусть x рабочих было в первом цехе. Во втором цехе было в три раза больше, чем в первом, поэтому количество рабочих во втором цехе можно обозначить через выражение 3x . В третьем цехе было на 15 рабочих меньше, чем во втором. Поэтому количество рабочих в третьем цехе можно обозначить через выражение 3x − 15 .

В задаче сказано, что всего рабочих было 685. Поэтому можно сложить выражения x , 3x , 3x − 15 и приравнять эту сумму к числу 685. В результате получим уравнение x + 3x + (3x − 15) = 685

Через переменную x было обозначено количество рабочих в первом цехе. Теперь мы нашли значение этой переменной, оно равно 100. Значит в первом цехе было 100 рабочих.

Во втором цехе было 3x рабочих, то есть 3 × 100 = 300 . А в третьем цехе было 3x − 15 , то есть 3 × 100 − 15 = 285

Ответ: в первом цехе было 100 рабочих, во втором — 300, в третьем — 285.

Задача 4. Две ремонтные мастерские в течение недели должны отремонтировать по плану 18 моторов. Первая мастерская выполнила план на 120%, а вторая — на 125%, поэтому в течение недели отремонтировали 22 мотора. Какой план по ремонту моторов на неделю имела каждая мастерская?

Решение

Пусть x моторов должна была отремонтировать первая мастерская. Тогда вторая мастерская должна была отремонтировать 18 − x моторов .

Поскольку первая мастерская выполнила свой план на 120%, это означает что она отремонтировала 1,2x моторов. А вторая мастерская выполнила свой план на 125%, значит она отремонтировала 1,25(18 − x ) моторов.

В задаче сказано, что было отремонтировано 22 мотора. Поэтому можно сложить выражения 1,2x и 1,25(18 − x ) , затем приравнять эту сумму к числу 22. В результате получим уравнение 1,2x + 1,25(18 − x ) = 22

Через переменную x было обозначено количество моторов, которые должна была отремонтировать первая мастерская. Теперь мы нашли значение этой переменной, она равна 10. Значит первая мастерская должна была отремонтировать 10 моторов.

А через выражение 18 − x было обозначено количество моторов, которые должна была отремонтировать вторая мастерская. Значит вторая мастерская должна была отремонтировать 18 − 10 = 8 моторов.

Ответ: первая мастерская должна была отремонтировать 10 моторов, а вторая — 8 моторов.

Задача 5. Цена товара повысилась на 30% и составляет теперь 91 руб. Сколько стоил товар до повышения цены?

Решение

Пусть x рублей стоил товар до повышения цены. Если цена увеличилась на 30% это означает, что она увеличилась на 0,30x рублей. После повышения цены товар начал стоить 91 руб. Сложим x с 0,30x и приравняем эту сумму к 91. В результате получим уравнение При уменьшении числа на 10% получилось 45. Найти первоначальное значение числа. x −

Ответ: чтобы получить 12%-й раствор соли, нужно к 1 кг 10%-го раствора добавить 0,25 кг 20%-го раствора.

Задача 12. Даны два раствора соли в воде, концентрации которых равны 20% и 30%. Сколько килограммов каждого раствора нужно смешать в одном сосуде, чтобы получить 25 кг 25,2%-го раствора?

Решение

Пусть x кг первого раствора нужно взять. Поскольку требуется приготовить 25 кг раствора, то массу второго раствора можно обозначить через выражение 25 − x.

В первом растворе будет содержáться 0,20x кг соли, а втором — 0,30(25 − x) кг соли. В полученном растворе содержание соли будет 25 × 0,252 = 6,3 кг. Сложим выражения 0,20x и 0,30(25 − x), затем приравняем эту сумму к 6,3. В результате получим уравнение

Значит первого раствора нужно взять 12 кг, а второго 25 − 12 = 13 кг.

Ответ: первого раствора нужно взять 12 кг, а второго 13 кг.

Понравился урок?
Вступай в нашу новую группу Вконтакте и начни получать уведомления о новых уроках

Свойства прямой в евклидовой геометрии.

Через любую точку можно провести бесконечно много прямых.

Через любые две несовпадающие точки можно провести единственную прямую.

Две несовпадающие прямые на плоскости или пересекаются в единственной точке, или являются

параллельными (следует из предыдущего).

В трёхмерном пространстве существуют три варианта взаимного расположения двух прямых:

  • прямые пересекаются;
  • прямые параллельны;
  • прямые скрещиваются.

Прямая линия — алгебраическая кривая первого порядка: в декартовой системе координат прямая линия

задается на плоскости уравнением первой степени (линейное уравнение).

Общее уравнение прямой.

Определение . Любая прямая на плоскости может быть задана уравнением первого порядка

Ах + Ву + С = 0,

причем постоянные А, В не равны нулю одновременно. Это уравнение первого порядка называют общим

уравнением прямой. В зависимости от значений постоянных А, В и С возможны следующие частные случаи:

. C = 0, А ≠0, В ≠ 0 - прямая проходит через начало координат

. А = 0, В ≠0, С ≠0 { By + C = 0} - прямая параллельна оси Ох

. В = 0, А ≠0, С ≠ 0 { Ax + C = 0} - прямая параллельна оси Оу

. В = С = 0, А ≠0 - прямая совпадает с осью Оу

. А = С = 0, В ≠0 - прямая совпадает с осью Ох

Уравнение прямой может быть представлено в различном виде в зависимости от каких - либо заданных

начальных условий.

Уравнение прямой по точке и вектору нормали.

Определение . В декартовой прямоугольной системе координат вектор с компонентами (А, В)

перпендикулярен прямой, заданной уравнением

Ах + Ву + С = 0.

Пример . Найти уравнение прямой, проходящей через точку А(1, 2) перпендикулярно вектору (3, -1).

Решение . Составим при А = 3 и В = -1 уравнение прямой: 3х - у + С = 0. Для нахождения коэффициента С

подставим в полученное выражение координаты заданной точки А. Получаем: 3 - 2 + C = 0, следовательно

С = -1. Итого: искомое уравнение: 3х - у - 1 = 0.

Уравнение прямой, проходящей через две точки.

Пусть в пространстве заданы две точки M 1 (x 1 , y 1 , z 1) и M2 (x 2, y 2 , z 2), тогда уравнение прямой ,

проходящей через эти точки:

Если какой-либо из знаменателей равен нулю, следует приравнять нулю соответствующий числитель. На

плоскости записанное выше уравнение прямой упрощается:

если х 1 ≠ х 2 и х = х 1 , если х 1 = х 2 .

Дробь = k называется угловым коэффициентом прямой .

Пример . Найти уравнение прямой, проходящей через точки А(1, 2) и В(3, 4).

Решение . Применяя записанную выше формулу, получаем:

Уравнение прямой по точке и угловому коэффициенту.

Если общее уравнение прямой Ах + Ву + С = 0 привести к виду:

и обозначить , то полученное уравнение называется

уравнением прямой с угловым коэффициентом k.

Уравнение прямой по точке и направляющему вектору.

По аналогии с пунктом, рассматривающим уравнение прямой через вектор нормали можно ввести задание

прямой через точку и направляющий вектор прямой.

Определение . Каждый ненулевой вектор (α 1 , α 2) , компоненты которого удовлетворяют условию

Аα 1 + Вα 2 = 0 называется направляющим вектором прямой.

Ах + Ву + С = 0.

Пример . Найти уравнение прямой с направляющим вектором (1, -1) и проходящей через точку А(1, 2).

Решение . Уравнение искомой прямой будем искать в виде: Ax + By + C = 0. В соответствии с определением,

коэффициенты должны удовлетворять условиям:

1 * A + (-1) * B = 0, т.е. А = В.

Тогда уравнение прямой имеет вид: Ax + Ay + C = 0, или x + y + C / A = 0.

при х = 1, у = 2 получаем С/ A = -3 , т.е. искомое уравнение:

х + у - 3 = 0

Уравнение прямой в отрезках.

Если в общем уравнении прямой Ах + Ву + С = 0 С≠0, то, разделив на -С, получим:

или , где

Геометрический смысл коэффициентов в том, что коэффициент а является координатой точки пересечения

прямой с осью Ох, а b - координатой точки пересечения прямой с осью Оу.

Пример . Задано общее уравнение прямой х - у + 1 = 0. Найти уравнение этой прямой в отрезках.

С = 1, , а = -1, b = 1.

Нормальное уравнение прямой.

Если обе части уравнения Ах + Ву + С = 0 разделить на число , которое называется

нормирующем множителем , то получим

xcosφ + ysinφ - p = 0 - нормальное уравнение прямой .

Знак ± нормирующего множителя надо выбирать так, чтобы μ * С < 0.

р - длина перпендикуляра, опущенного из начала координат на прямую,

а φ - угол, образованный этим перпендикуляром с положительным направлением оси Ох.

Пример . Дано общее уравнение прямой 12х - 5у - 65 = 0 . Требуется написать различные типы уравнений

этой прямой.

Уравнение этой прямой в отрезках :

Уравнение этой прямой с угловым коэффициентом : (делим на 5)

Уравнение прямой :

cos φ = 12/13; sin φ= -5/13; p = 5.

Следует отметить, что не каждую прямую можно представить уравнением в отрезках, например, прямые,

параллельные осям или проходящие через начало координат.

Угол между прямыми на плоскости.

Определение . Если заданы две прямые y = k 1 x + b 1 , y = k 2 x + b 2 , то острый угол между этими прямыми

будет определяться как

Две прямые параллельны, если k 1 = k 2 . Две прямые перпендикулярны,

если k 1 = -1/ k 2 .

Теорема .

Прямые Ах + Ву + С = 0 и А 1 х + В 1 у + С 1 = 0 параллельны, когда пропорциональны коэффициенты

А 1 = λА, В 1 = λВ . Если еще и С 1 = λС , то прямые совпадают. Координаты точки пересечения двух прямых

находятся как решение системы уравнений этих прямых.

Уравнение прямой, проходящей через данную точку перпендикулярно данной прямой.

Определение . Прямая, проходящая через точку М 1 (х 1 , у 1) и перпендикулярная к прямой у = kx + b

представляется уравнением:

Расстояние от точки до прямой.

Теорема . Если задана точка М(х 0 , у 0), то расстояние до прямой Ах + Ву + С = 0 определяется как:

Доказательство . Пусть точка М 1 (х 1 , у 1) - основание перпендикуляра, опущенного из точки М на заданную

прямую. Тогда расстояние между точками М и М 1 :

(1)

Координаты x 1 и у 1 могут быть найдены как решение системы уравнений:

Второе уравнение системы - это уравнение прямой, проходящей через заданную точку М 0 перпендикулярно

заданной прямой. Если преобразовать первое уравнение системы к виду:

A(x - x 0) + B(y - y 0) + Ax 0 + By 0 + C = 0,

то, решая, получим:

Подставляя эти выражения в уравнение (1), находим:

Теорема доказана.